Site Loader

Содержание

Формула силы Лоренца в физике

Содержание:

Определение и формула силы Лоренца

Определение

Сила $\bar{F}$ , действующая на движущуюся заряженную частицу в магнитном поле, равная:

$$\bar{F}=q[\bar{v} \times \bar{B}](1)$$

называется силой Лоренца (магнитной силой).

Исходя из определения (1) модуль рассматриваемой силы:

$$F=q v B \sin \alpha(2)$$

где $\bar{v}$ – вектор скорости частицы, q – заряд частицы, $\bar{B}$ – вектор магнитной индукции поля в точке нахождения заряда, $\alpha$ – угол между векторами $\bar{v}$ и $\bar{B}$. Из выражения (2) следует, что если заряд движется параллельно силовым линиям магнитного поля,то сила Лоренца равна нулю. Иногда силу Лоренца стараясь выделить, обозначают, используя индекс: $\bar{F}_L$

Направление силы Лоренца

Сила Лоренца (как и всякая сила) – это вектор. Ее направление перпендикулярно вектору скорости $\bar{v}$ и вектору $\bar{B}$ (то есть перпендикулярно плоскости, в которой находятся векторы скорости и магнитной индукции) и определяется правилом правого буравчика (правого винта) рис.

{2}}}}$ – релятивистский множитель Лоренца, c – скорость света в вакууме.

Сила Лоренца — это центростремительная сила. По направлению отклонения элементарной заряженной частицы в магнитном поле делают вывод о ее знаке (рис.2).

Формула силы Лоренца при наличии магнитного и электрического полей

Если заряженная частица перемещается в пространстве, в котором находятся одновременно два поля (магнитное и электрическое), то сила, которая действует на нее, равна:

$$\bar{F}=q \bar{E}+q[\bar{v} \times \bar{B}](4)$$

где $\bar{E}$ – вектор напряженности электрического поля в точке, в которой находится заряд. Выражение (4) было эмпирически получено Лоренцем. Сила $\bar{F}$, которая входит в формулу (4) так же называется силой Лоренца (лоренцевой силой). Деление лоренцевой силы на составляющие: электрическую $(\bar{F} = q \bar{E})$ и магнитную $(\bar{F}=q[\bar{v} \times \bar{B}])$ относительно, так как связано с выбором инерциальной системы отсчета. Так, если система отсчета будет двигаться с такой же скоростью $\bar{v}$, как и заряд, то в такой системе сила Лоренца, действующая на частицу, будет равна нулю.{2}}{R}(1.4)$$

Из выражения (1.3) получим скорость:

$$v=\frac{q B R}{m}(1.5)$$

Период обращения электрона по окружности можно найти как:

$$T=\frac{2 \pi R}{v}=\frac{2 \pi m}{q B}(1.6)$$

Зная период, можно найти угловую скорость как:

$$\omega=\frac{2 \pi}{T}=\frac{q_{e} B}{m}$$

Ответ. $\omega=\frac{q_{e} B}{m}$

Слишком сложно?

Формула силы Лоренца не по зубам? Тебе ответит эксперт через 10 минут!

Пример

Задание. Заряженная частица (заряд q, масса m) со скоростью vвлетает в область, где имеется электрическое поле напряженностью E и магнитное поле с индукцией B. Векторы $\bar{E}$ и $\bar{B}$ совпадают по направлению. Каково ускорение частицы в моментначалаперемещения в полях, если $\bar{v} \uparrow \bar{B} \uparrow \bar{E}$?

Решение.

Сделаем рисунок.

На заряженную частицу действует сила Лоренца:

$$\bar{F}=q \bar{E}+q[\bar{v} \times \bar{B}](2.1)$$

Магнитная составляющая имеет направление перпендикулярное вектору скорости ($\bar{v}$) и вектору магнитной индукции ($\bar{B}$). Электрическая составляющая сонаправлена с вектором напряжённости ($\bar{E}$) электрического поля. В соответствии со вторым законом Ньютона имеем:

$$\bar{F}=q \bar{E}+q[\bar{v} \times \bar{B}]=m \bar{a}(2.2)$$

Получаем, что ускорение равно:

$$\frac{q \bar{E}+q[\bar{v} \times \bar{B}]}{m}=\bar{a}(2.3)$$

Если скорость заряда параллельна векторам $\bar{E}$ и $\bar{B}$, тогда $[\bar{v} \times \bar{B}]=0$, получим:

$$\bar{a}=\frac{q \bar{E}}{m}$$

Ответ. $\bar{a}=\frac{q \bar{E}}{m}$

Читать дальше: Формула силы натяжения нити.

Урок 3. магнитная индукция. действие магнитного поля на проводник с током и движущуюся заряженную частицу — Физика — 11 класс

Физика, 11 класс

Урок 3. Магнитная индукция. Действие магнитного поля на проводник и движущуюся заряжённую частицу

Перечень вопросов, рассматриваемых на уроке:

1) магнитное поле;

2) вектор магнитной индукции, линии магнитной индукции;

3) сила Ампера, сила Лоренца;

4) правило буравчика, правило левой руки.

Глоссарий по теме

Магнитная индукция – векторная величина, характеризующая величину и направление магнитного поля.

Сила Ампера – сила, действующая со стороны магнитного поля на проводник с током.

Сила Лоренца – сила, действующая со стороны магнитного поля на движущую частицу с зарядом.

Правило «буравчика» — правило для определения направления магнитного поля проводника с током.

Правило левой руки – правило для определения направления силы Ампера и силы Лоренца.

Соленоид – проволочная катушка.

Рамка с током – небольшой длины катушка с двумя выводами из скрученного гибкого проводника с током, способная поворачиваться вокруг оси, проходящей через диаметр катушки.

Основная и дополнительная литература по теме урока

Мякишев Г.Я., Буховцев Б.Б.,. Чаругин В.М. Физика.11 класс. Учебник для общеобразовательных организаций. М.: Просвещение, 2014. – С. 3 – 20

2. А.П. Рымкевич. Сборник задач по физике. 10-11 классы. — М: Дрофа, 2009. – С.109 — 112

Основное содержание урока

Магнитное поле – особый вид материи, которая создаётся электрическим током или постоянными магнитами. Для демонстрации действия и доказательства существования магнитного поля служат магнитная стрелка, способная вращаться на оси, или небольшая рамка (или катушка) с током, подвешенная на тонких скрученных гибких проводах.

Рамка с током и магнитная стрелка под действием магнитного поля поворачиваются так, что северный полюс (синяя часть) стрелки и положительная нормаль рамки указывают направление магнитного поля.

Магнитное поле, созданное постоянным магнитом или проводником с током, занимает всё пространство в окрестности этих тел. Магнитное поле принято (удобно) изображать в виде линий, которые называются линиями магнитного поля. Магнитные линии имеют вихревой характер, т.е. линии не имеют ни начала, ни конца, т.е. замкнуты. Направление касательной в каждой точке линии совпадает с направлением вектора магнитной индукции. Поля с замкнутыми линиями называются вихревыми.

Магнитное поле характеризуется векторной величиной, называемой магнитной индукцией. Магнитная индукция характеризует «силу» и направление магнитного поля – это количественная характеристика магнитного поля.

Она обозначается символом За направление вектора магнитной индукции принимают направление от южного полюса к северному магнитной стрелки, свободно установившейся в магнитном поле.

Направление магнитного поля устанавливают с помощью вектора магнитной индукции.

Направление вектора магнитной индукции прямого провода с током определяют по правилу буравчика (или правого винта).

Правило буравчика звучит следующим образом:

если направление поступательного движения буравчика совпадает с направлением тока в проводнике, то направление вращения ручки буравчика совпадает с направлением линий магнитного поля тока.

Направление магнитного поля внутри соленоида определяют по правилу правой руки.

Определим модуль вектора магнитной индукции.

Наблюдения показывают, что максимальное значение силы, действующей на проводник, прямо пропорционально силе тока, длине проводника, находящегося в магнитном поле.

F_max ~ I; F ~ Δl.

Тогда, зависимость силы от этих двух величин выглядит следующим образом

Отношение зависит только от магнитного поля и может быть принята за характеристику магнитного поля в данной точке.

Величина, численно равная отношению максимальной силы, действующей на проводник с током, на произведение силы тока и длины проводника, называется модулем вектора магнитной индукции:

Единицей измерения магнитной индукции является 1 тесла (Тл).

1Тл = 1Н/(1А∙1м).

Закон Ампера:

Сила, действующая на проводник с током в магнитном поле, равна произведению модуля магнитной индукции, силы тока, длины проводника и синуса угла между вектором магнитной индукции и направлением тока:

где α – угол между вектором B и направлением тока.

Направление силы Ампера определяется правилом левой руки:

Если ладонь левой руки развернуть так, чтобы линии магнитной индукции входили в ладонь, а четыре вытянутых пальца были направлены по направлению тока, то отогнутый на 900 большой палец покажет направление силы Ампера.

Сила Ампера — сила, действующая на проводник с током со стороны магнитного поля.

Сила Лоренца – сила, действующая на движущуюся заряженную частицу со стороны магнитного поля. Её численное значение равно произведению заряда частицы на модули скорости и магнитной индукции и синус угла меду векторами скорости и магнитной индукции:

– заряд частицы;

– скорость частицы;

B – модуль магнитной индукции;

– угол между векторами скорости частицы и магнитной индукции.

Направление силы Лоренца также определяют по правилу левой руки:

Если четыре вытянутых пальца левой руки направлены вдоль вектора скорости заряженной частицы, а вектор магнитной индукции направлен в ладонь, то отведённый на 900 большой палец покажет направление силы Лоренца. Если частица имеет заряд отрицательного знака, то направление силы Лоренца противоположно тому направлению, которое имела бы положительная частица.

Получим формулы для радиуса окружности и периода вращения частицы, которая влетает в однородное магнитное поле перпендикулярно линиям магнитной индукции, применяя формулы второго закона Ньютона и центростремительного ускорения.

Согласно 2-му закону Ньютона

Отсюда

Время, за которое частица делает полный оборот (период обращения), равно:

Многим юным бывает досадно, что они не родились в старые времена, когда делались открытия. Им кажется, что теперь всё известно и никаких открытий на их долю не осталось.

Одной из нераскрытых тайн является механизм земного магнитного поля. Как же и чем вызывается магнитное поле Земли? Подумайте и может быть…

Одна из возможных гипотез.

Как известно, ядро Земли имеет высокую температуру

и высокую плотность. Судя по исследованиям, в самом центре содержится твёрдое ядро. При вращении Земли вокруг своей оси центр тяжести не совпадает с геометрическим центром из-за притяжения Солнца. В результате сместившееся из центра ядро вращаясь относительно оболочки Земли вызывает такое же движение жидкой расплавленной массы мантии, как чайная ложка, перемешивающая воду в стакане. Получается не что иное, как направленное движение зарядов. Есть электрический ток, а он, в свою очередь, создаёт магнитное поле.

Разбор тренировочных заданий

1. На рисунке изображён проводник с током, помещённый в магнитное поле. Стрелка указывает направление тока в проводнике. Вектор магнитной индукции направлен перпендикулярно плоскости рисунка к нам. Как направлена сила, действующая на проводник с током?

Варианты ответов:

1. вправо →;

2. влево ←;

3. вниз ↓;

4. вверх ↑.

— точка означает, что магнитная индукция направлена на нас из глубины плоскости рисунка.

Используя правило левой руки, определяем направление силы Ампера:

Левую руку располагаем так, чтобы линии магнитной индукции входили в ладонь, 4 пальца направим вниз по направлению тока, тогда отогнутый на 900 большой палец покажет направление силы Ампера, т. е. она направлена влево.

Правильный вариант:

2. влево ←.

2. По проводнику длиной 40 см протекает ток силой 10 А. Чему равна индукция магнитного поля, в которое помещён проводник, если на проводник действует сила 8 мН?

(Ответ выразите в мТл).

3. Определите модуль силы, действующей на проводник длиной 50 см при силе тока 10 А в магнитном поле с индукцией 0,15 Тл. (Ответ выразите в мН).

4. Протон в магнитном поле с индукцией 0,01 Тл описал окружность радиусом 10 см. Найдите скорость протона. (Ответ выразите в км/с, округлив до десятков)

5. С какой скоростью влетает электрон в однородное магнитное поле (индукция 1,8 Тл) перпендикулярно к линиям индукции, если магнитное поле действует на него с силой 3,6∙10¹² Н? Ответ выразите в км/с.

6. Электрон движется в однородном магнитном поле с индукцией 3,14мТл. Чему равен период обращения электрона? (Ответ выразите в наносекундах, округлив до целых)

2. Дано:

l = 40cм = 0,4 м,

I = 10 A,

F =8 мН = 0,008 Н.

Найти: B

Решение:

Запишем формулу модуля магнитной индукции:

Делаем расчёт:

B = 0,008 Н / ( 0,4м·10 A) = 0,002 Tл = 2 мTл.

Ответ: 2 мTл.

3. Дано:

l = 50 cм = 0,5 м,

I = 10 A,

B = 0,l5 Tл.

Найти: F

Решение:

Запишем формулу силы Ампера:

Делаем расчёт:

F = 0,l5 Tл· 10 A· 0,5 м = 0,75 Н = 750 мН

Ответ: 750 мН.

4. Дано:

B = 0,0l Tл,

r = l0 cм = 0,l м.

Найти: v

Решение:

Заряд протона равен: q₀ = l,6·l0⁻ˡ⁹ Кл,

масса протона: m = l,67·l0⁻²⁷ кг.

Согласно 2-му закону Ньютона:

Отсюда следует:

Делаем расчёт:

v = ( l,6·l0⁻ˡ⁹ Кл·0,l м·0,0l Tл) / l,67·l0⁻²⁷ кг ≈ 0,00096·l0⁸ м/с ≈ l00 км/с.

Ответ: v ≈ l00 км/с.

5. Дано:

B = l,8 Tл,

F = 3,6·l0⁻¹² Н,

α = 90°.

Найти:

Решение:

Заряд электрона равен: q₀ = l,6·l0⁻ˡ⁹ Кл.

Используем формулу силы Лоренца:

.

Выразим из формулы силы скорость, учитывая, что sin90°=l,

Делаем расчёт:

v = 3,6·l0⁻¹² Н / (l,6·l0⁻ˡ⁹ Кл· l,8 Tл) = l,25·l0⁷м/с = l2500 км/с.

Ответ: v = l2500 км/с.

6. Дано:

B = 3,l4 мТл = 3,l4·l0⁻³ Tл,

q₀ = l,6·l0⁻ˡ⁹ Кл,

Найти: Т

Решение:

Масса электрона равна: m = 9,l·l0⁻³¹ кг.

Время, за которое частица делает полный оборот (период обращения), равно:

Делаем расчёт:

T = 2·3,l4·9,l·l0⁻³¹ кг/( l,6·l0⁻ˡ⁹ Кл·3,l4·l0⁻³ Tл) = ll,375·l0⁻⁹ с ≈ ll нс.

Ответ: T ≈ ll нс.

Сила Лоренца и ее воздействие на электрический заряд

Электрические заряды, движущиеся в определенном направлении, создают вокруг себя магнитное поле, скорость распространения которого в вакууме равно скорости света, а в других средах чуть меньше. Если движение заряда происходит во внешнем магнитном поле, то между внешним магнитным полем и магнитным полем заряда возникает взаимодействие. Так как электрический ток – это направленное движение заряженных частиц, то сила, которая будет действовать в магнитном поле на проводник с током, будет являться результатом отдельных (элементарных) сил, каждая из которых прикладывается к элементарному носителю заряда.

Процессы взаимодействия внешнего магнитного поля и движущихся зарядов исследовались Г. Лоренцом, который в результате многих своих опытов вывел формулу для расчета силы, действующей на движущуюся заряженную частицу со стороны магнитного поля. Именно поэтому силу, которая действует на движущийся в магнитном поле заряд, называют силой Лоренца.

Сила, действующая на проводник стоком (из закона Ампера), будет равна:

По определению сила тока равна I = qn (q – заряд, n – количество зарядов, проходящее через поперечное сечение проводника за 1 с). Отсюда следует:

Где: n0 – содержащееся в единице объема количество зарядов, V – их скорость движения, S – площадь поперечного сечения проводника. Тогда:

Подставив данное выражение в формулу Ампера, мы получим:

Данная сила будет действовать на все заряды, находящиеся в объеме проводника: V = Sl. Количество зарядов, присутствующих в данном объеме будет равно:

Тогда выражение для силы Лоренца будет иметь вид:

Отсюда можно сделать вывод, что сила Лоренца, действующая на заряд q, который двигается в магнитном поле, пропорциональна заряду, магнитной индукции внешнего поля, скорости его движения и синусу угла между V и В, то есть:

За направление движения заряженных частиц принимают направление движения положительных зарядов. Поэтому направление данной силы может быть определено с помощью правила левой руки.

Сила, действующая на отрицательные заряды, будет направлена в противоположную сторону.

Сила Лоренца всегда направлена перпендикулярно скорости V движения заряда и поэтому работу она не совершает. Она изменяет только направление V, а кинетическая энергия и величина скорости заряда при его движении в магнитном поле остаются неизменными.

Когда заряженная частица движется одновременно в магнитном и электрическом полях, на него будет действовать сила:

Где Е – напряженность электрического поля.

Рассмотрим небольшой пример:

Электрон, прошедший ускоряющую разность потенциалов 3,52∙103 В, попадает в однородное магнитное поле перпендикулярно линиям индукции. Радиус траектории r = 2 см, индукция поля 0,01 Т. Определить удельный заряд электрона.

Решение:

Удельный заряд – это величина, равная отношению заряда к массе, то есть e/m.

В магнитном поле с индукцией В на заряд, движущийся со скоростью V перпендикулярно линиям индукции, действует сила Лоренца FЛ = BeV. Под ее действием заряженная частица будет перемещаться по дуге окружности. Так как при этом сила Лоренца вызовет центростремительное ускорение, то согласно 2-му закону Ньютона можно записать:

Кинетическую энергию, которая будет равна mV2/2, электрон приобретает за счет работы А сил электрического поля (А = eU), подставив в уравнение получим:

Преобразовав эти соотношения и исключив из них скорость, получим формулу для определения удельного заряда электрона:

Подставив исходные данные, выраженные в СИ, получим:

Проверяем размерность:

И кому интересно — видео о движении заряженных частиц:

Сила Лоренца — Справочник химика 21

    В масс-спектрометре (рис. 5) молекулы в высоком вакууме бомбардируются потоком электронов, которые выбивают из них собственные электроны, превращая в положительно заряженные ионы. Пучок таких ионов далее ускоряется электрическим полем и попадает в магнитное поле. При этом за счет силы Лоренца ионы отклоняются от прямолинейного движения. Сила Лоренца зависит от заряда иона Q, скорости его движения, а ускорение, отклоняющее пучок ионов, определяется и массой ионов. В итоге угол отклонения зависит от отношения Q/m и оказывается разным для частиц разной массы. Если, например, в пучке имелись молекулы Ю2, и Ю2, [c.27]
    Здесь и, V, w — компоненты скорости W. Если на заряды действует также электрическое поле, то к силе Лоренца добавится сила Кулона, которая, согласно (2), равна еЕ. Полная электромагнитная сила, действующая на заряд, будет в этом случае [c.190]

    Поле соленоида и тороида. Сила Лоренца. Движение заряженных частиц в электрическом и магнитном полях. Ускорители заряженных частиц — их применение в производстве. [c.165]

    Распространение света в веществе с точки зрения классической теории связано с осцилляцией электронов в атомах и молекулах, которую вызывает падающий свет. Электромагнитная волна света, как указывалось, представляет систему двух взаимно перпендикулярных полей электрического и магнитного. Обычно для задачи распространения света в веществе рассматривают только электрическую компоненту электромагнитной волны, так как сила Лоренца, действующая на электрон со стороны магнитного поля, равна е [ухВ], где V —скорость электрона, В —магнитная индукция. Эта сила мала из-за малой величины и/с ( Го=сБо)- [c.175]

    В =Вг. Направление распространения света выбирают за ось г. Вектор напряженности электрического поля световой волны ё находится в перпендикулярной плоскости ху (рис. Х1У.З). Если не учитывать затухания, то дифференциальное уравнение колеблющегося электрона можно записать в следующем виде с использованием выражения для силы Лоренца  [c.250]

    При переходе к области поглощения в эксперименте по эффекту Фарадея необходимо учитывать эффект Зеемана — расщепление спектральных линий испускания и поглощения в магнитном поле. Согласно упрощенной схеме эффекта Зеемана влияние магнитного поля в направлении г состоит в том, что колеблющиеся в плоскости ху электроны можно рассматривать как вращающиеся по и против часовой стрелки ( рис. Х1У.4). Однако сила Лоренца = = —е[уХВ] = — [гХВ] будет изменять частоту вращения электронов. Для левого круга частота увеличивается, поскольку сила / л направлена в центр (правило правой руки) и [c.253]

    Влияние магнитного поля Н можно учесть, заменив правую часть уравнения (176) силой Лоренца  [c.132]

    В магнитном поле на электрон, помимо силы /,,, действует сила Лоренца д = (е/с) vH, ко- торая, согласно рис. 126, а, на-правлена в сторону, противоположную fe- Под действием этой силы скорость электрона изменяется и становится равной v = or соответственно центростремительная сила становится равной [c.294]

    Если через волновод в виде стальной проволоки диаметром В распространяется поперечная ультразвуковая волна с волновым вектором qz (ось 2 — вдоль стержня), то, вследствие возникновения областей сжатия и растяжения в металле волновода, появляется переменное электрическое поле. Причина его появления связана со смещением ионов в узлах кристаллической решетки металла. При этом электроны практически не взаимодействуют с ультразвуком и движутся только под действием электрического поля ионов. Фактически возникают микротоки. Если приложить магнитное поле перпендикулярно смещению ионов, то под действием силы Лоренца электроны начнут отклоняться в плоскости, перпендикулярной магнитному полю, а само направление отклонения электронов определяется по правилу левой руки. Этот эффект называется магнитоакустическим эффектом. При соответствующем подборе размеров диаметра проволоки О волновода, величины магнитного поля В поплавка, и длины ультразвуковой волны траектория замкнется и по поверхности проволоки волновода будет протекать электрический ток. [c.83]


    Вода всегда содержит ионы солей, которые, двигаясь в магнитном поле, испытывают действие силы Лоренца — сипы взаимодействия магнитного поля и движущейся в нем заряженной частицы, В результате происходит поляризация ионов, их деформация — магнитное поле как бы сортирует ионы по знаку и величине заряда, что повышает вероятность сближения поляризованных ионов и образования центров кристаллизации. [c.31]

    В основе представлений об электромагнитных явлениях и свойствах материи лежит несколько фундаментальных законов, являющихся обобщением экспериментальных фактов. Это закон Кулона, сила Лоренца, законы электромагнитной индукции. [c.645]

    В качестве соотношения, определяющего смысл и количественную меру интенсивности магнитного поля— магнитной индукции В, — можно использовать выражение для силы Лоренца Г. Эта сила, согласно формуле (3.9.46), действует только на движущийся заряд Q и зависит от скорости его движения V  [c.653]

    Сила Лоренца, действующая на электроны в единице объема среды, равна  [c.88]

    Эти способы, называемые также магнитоиндукционными, основаны на так называемых силах Лоренца.. Имеется в виду сила F, действующая на заряд е, движущийся со скоростью v в магнитном поле с индукцией В [28, 580]. Для нее справедливо соотношение [c.171]

    Подзаряженные ионы попадают затем в магнитное поле, где на них действуют силы Лоренца. Эти силы отклоняют и закручивают траектории движения ионов. За один цикл из воды можно удалить 90% ионов электролитов. Схема установки магнитного обессоливания воды показана на рис. 47. [c.194]

    Преобразователи Холла работают по принципу возникновения ЭДС в результате искривления пути носителей тока в металлах и полупроводниках, находящихся в магнитном поле под действием силы Лоренца. [c.333]

    Будем считать, что отсутствует внешнее постоянное магнитное поле. Будем также пренебрегать влиянием переменного магнитного поля. Поскольку отношение высокочастотного магнитного поля к электрическому по порядку величины равно е/соА., где с — скорость света, а А. — характерное расстояние изменения высокочастотного поля, то вкладом такого магнитного поля в силу Лоренца можно пренебречь, если [c.142]

    При магнитной обработке возникновение электромагнитных полей является следствием перемещения воды в магнитном поле. В этих условиях возникает противоположное циклотронное перемещение катионов и анионов под действием сил Лоренца, обусловливающих движением ионов вокруг силовых линий магнитного поля с определенной частотой. Этот процесс рассмотрен в работах [51, 124] и др., а также Г. А. Семеновым [19, с. 37—40]. [c.103]

    Многие предварительные расчеты показывают, что действие сил Лоренца на ионы раствора, перемещающегося в магнитном поле, очень невелики. В то же время есть много прямых и косвенных экспериментальных доказательств того, что при этом изменение характера движения ионов весьма заметно. Мы уже упоминали об установленном (хотя и незначительном) эффекте Холла. В гидродинамике известен эффект Гартмана при протекании токопроводящей вязкой жидкости между полюсами магнита профиль скоростей потока меняется — параболическая форма профиля искажается. Эти противоречия могут быть в известной мере объяснены тем, что в условиях магнитной обработки, как уже неоднократно отмечалось, на ионы действует не только магнитное, но и электрическое поле, индуцируемое в перемещающейся электропроводящей жидкости. [c.103]

    Под действием силы Лоренца происходит заметное изменение траектории электронов, если скорость невелика, т. е. вторичных электронов (рис. 22.15). Домены с противоположным направлением вектора индукции В будут иметь различную яркость Разрешение составляет обычно несколько микрометров. [c.563]

    Функция Гамильтона (11.15), а следовательно, и гамильтониан (И.16) получаются в том случае, когда силы, действующие на частицу, не зависят от скорости. Однако в электромагнитном поле на движущийся заряд действует сила Лоренца [c.44]

    В работе [3] рассмотрен один из вариантов плазменной центрифуги также для разделения изотопов урана. Врап] епие плазмы осуп] еств-ляют и регулируют силой Лоренца. Урановая плазма создается при инициировании электрической дуги между электродами с различными радиусами. Скорость врап] ения для Нз 10 см/с. Фактор разделения а и работа разделения сильно зависят от W p (работа разделения пропорциональна W ), температуры и плотности плазмы. [c.472]

    Если читатель не специалист по приборостроению, задача может показаться не вполне понятной. Но суть дела проста. В магнитном поле расположена легкая рамка, от малейшего сотрясения она колеблется — с этим надо бороться. Соль задачи — во множестве ограничений нельзя усложнять прибор, нельзя утяжелять рамку, нельзя применять жидкостное и магнитоиндукционное демпфирование… Дана невепольная система есть вещество (рамка) и магнитное поле, не взаимодействующие между собой. Ответ очевиден. Надо привязать к рамке второе вещество, которое будет взаимодействовать с магнитным полем. Такое вещество — движущиеся заряды. На боковые поверхности рамки наносят электрет при колебаниях, т. е. при движении рамки в магнитном поле, позникает сила Лоренца, пропорциональная скорости перемещения зарядов и гасящая колебания (а. с. 481844). [c.114]

    Магнитные свойства металлов связаны с их электрическими свойствами, поскольку элементарные носители магнетизма — электроны — обладают как магнитным моментом, так и элеюрическим зарядом. Наряду с общими для всех твердых тел элеюрическими свойствами магнитные материалы обладаюг рядом специфических электрических свойств, зависящих от самопроизвольной намагниченности. В магнитных материалах в каждом ферромагнитном домене на электрон проводимости даже при нулевом внешнем магнитном поле действует сила Лоренца. [c.17]


    Сила Д , называемая силой Лоренца, перпендикулярна к плоскости, в которой лежат векторы W и В для положительного заряда она определяется по правилу левой руки. Если W J В, то сила имеет наибольшее значение (sin а = 1), если W II В, то сила равна нулю (sina = 0). [c.190]

    После этого пучок попадает ь маг П]тный анализатор, где на него действует магнитное поле с иидукщ1ен В, также перпендикулярное направлению пучка. Под действием силы Лореииа evB пучок снова искривляется, причем радиус кривизны определяется равенством силы Лоренца и центробежной силы  [c.45]

    Я II /) магнитные поля не должны были бы влиять на сопротивление. Действительно, если скорость у всех электронов одна и та же, то в стационарном состоянии действие силы Лоренца и электрического поля Холла полностью компенсируются и пути электронов отстаются неискаженными магнитным полем, а сопротивление — неизменным. [c.331]

    Сопротивление образца изменяется благодаря максвеллов-кому распределению скоростей электронов если поле Холла компенсирует отклонение магнитным полем для электронов некоторой средней скорости, то электроны со скоростью меньше средней будут отклоняться в сторону электрической силы Холла еЕу, а электроны со скоростью больше средней будут отклоняться в сторону магнитной силы Лоренца еУхН с. Это ведет к уменьшению длины свободного пробега и тех, и других электронов в направлении внешнего электрического поля Е , а следовательно, и к росту сопротивления. [c.331]

    Движение электрона по замкнутой орбите вокруг ядра можно уподобить электрическому току. Такой замкнутый ток создает определенный магнитный момент. При помещении вещества в магнитное поле Н на вращающийся электрон действует дополнительная сила (сила Лоренца), которая изменяет частоту обращения Электрона по орбите, т. е. создает добавочный круговой ток и соответствующий дополнительный магнитный момент, направленный противоположно полю Н. Вследствие этого диамагнитное вещество выталкивается из поля. Такому дополнительному магнитному моменту соответствует диамагнитная воспри-11мчн8ость. имеющая отрицательный знак. Суммируя эти дополнительные магнит- [c.300]

    Здесь Uij — потенциальная энергия парного 1 чаимодойстния, а i j — внешняя сила, действующая па г-ю частицу эта сила для случая заряженных частиц может быть силой Лоренца (44.12). [c.186]

    Сила Лоренца Ра определяется впешпими электрическим Е и магнитным В полями, а также самосогласоваппым электрическим полем, обусловленным кулоновским взаимодействием частиц В нашем изложении мы полностью опустим эффекты, связанные с вихревым самосогласованным полем токов плазмы, которое не может быть рассмотрено при учете лишь кулоновского взаимодействия ). [c.233]

    С помощью формулы (П.П.9) —(П.II.11) получается следующая формула для спектральпоп плотпости флуктуаций силы Лоренца, дейстнующеи на частицу сорта  [c.311]

    Аг, Кг и др.) [68]. Улавливание радиоактивных частиц из воздушного потока можно осуществлять магнитными ловушками [68]. В магнитном поле заряженные частицы пыли, содержащие радионуклиды, отклоняются под действием силы Лоренца от первоначального направления движения и поступают в пьшесборники. [c.214]

    Между тем, как отмечает В. И. Миненко [12, с. 17— 18], воздействие магнитного поля на жидкость может вызывать меньшие изменения, чем действие электрических сил. Так, силы Лоренца, возникающие при течении воды и действующие перпендикулярно направлению потока, за время пребывания воды в поле (0,1 с) при градиенте электрического поля порядка десятков милливольт на 1 см способны произвести над одним грамм-ионом работу, измеряемую несколькими сотнями джоулей. Возможно энергия перекачивания жидкости является некоторым добавочным источником изменения изобарно-изотермического потенциала системы при ее магнитной обработке. Следует также рассмотреть и учесть возможное взаимодействие с магнитным полем растворенного в воде парамагнитного кислорода. [c.92]

    По выходе из выходной щели ионного источника мо-ноэнергетический ионный пучок в виде ленты, поставленной на ребро по отношению к плоскости рис. 1-2, попадает в поперечное магнитное поле. Здесь коны начинают испытывать силу Лоренца со стороны магнитного поля, превращающую их прямолинейную траекторию в окружности различных радиусов, так как ее можно считать центростремительной силой. Для центростремительной силы справедливо соотношение [c.12]

    Пояснение. Приравняйте силу Лоренца qvHI v —-скорость электронов) силе в поле эдс Холла —qV] вспомните, что плотность тока есть произведение скорости электронов на заряд, переносимый ими. [c.88]


Сила Лоренца

Сила Лоренца. Определение и формула

Определение 1

Сила Ампера, воздействующая на часть проводника длиной Δl с некоторой силой тока I, находящийся в магнитном поле B, F=I·B·Δl·sin α может выражаться через действующие на конкретные носители заряда силы.

Пускай заряд носителя обозначается как q, а n является значением концентрации носителей свободного заряда в проводнике. В этом случае произведение n·q·υ·S, в котором S представляет собой площадь поперечного сечения проводника, эквивалентно току, протекающему в проводнике, а υ – это модуль скорости упорядоченного движения носителей в проводнике:

I=q·n·υ·S.

Определение 2

Формула силы Ампера может записываться в следующем виде:

F=q·n·S·Δl·υ·B·sin α.

По причине того, что полное число N носителей свободного заряда в проводнике сечением S и длиной Δl равняется произведению n·S·Δl, действующая на одну заряженную частицу сила равняется выражению: FЛ=q·υ·B·sin α.

Найденная сила носит название силы Лоренца. Угол α в приведенной формуле эквивалентен углу между вектором магнитной индукции B→ и скоростью ν→.

Направление силы Лоренца, которая воздействует частицу с положительным зарядом, таким же образом, как и направление силы Ампера, находится по правилу буравчика или же с помощью правила левой руки. Взаимное расположение векторов ν→, B→ и FЛ→ для частицы, несущей положительный заряд, проиллюстрировано на рис. 1.18.1.

Рисунок 1.18.1. Взаимное расположение векторов ν→, B→ и FЛ→. Модуль силы Лоренца FЛ→ численно эквивалентен произведению площади параллелограмма, построенного на векторах ν→ и B→ и заряда q.

Сила Лоренца направлена нормально, то есть перпендикулярно, векторам ν→ и B→.

Сила Лоренца не совершает работы при движении несущей заряд частицы в магнитном поле. Данный факт приводит к тому, что модуль вектора скорости в условиях движения частицы так же не меняет своего значения.

Если заряженная частица движется в однородном магнитном поле под действием силы Лоренца, а ее скорость ν→ лежит в плоскости, которая направлена нормально по отношению к вектору B→, то частица будет совершать движение по окружности некоторого радиуса, рассчитывающегося с помощью следующей формулы:

R=mνqB.

Сила Лоренца в данном случае применяется в качестве центростремительной силы (рис. 1.18.2).

Рисунок 1.18.2. Круговое движение заряженной частицы в однородном магнитном поле.

Для периода обращения частицы в однородном магнитном поле будет справедливо следующее выражение:

T=2πRυ=2πmqB.

Данная формула наглядно демонстрирует отсутствие зависимости заряженных частиц заданной массы m от скорости υ и радиуса траектории R.

Применение силы Лоренца

Определение 3

Приведенное снизу соотношение представляет собой формулу угловой скорости движения заряженной частицы, происходящего по круговой траектории:

ω=υR=υqBmυ=qBm.

Оно носит название циклотронной частоты. Данная физическая величина не имеет зависимости от скорости частицы, из чего можно сделать вывод, что и от ее кинетической энергии она не зависит.

Определение 4

Данное обстоятельство находит свое применение в циклотронах, а именно в ускорителях тяжелых частиц (протонов, ионов).

На рисунке 1.18.3 приводится принципиальная схема циклотрона.

Рисунок 1.18.3. Движение заряженных частиц в вакуумной камере циклотрона.

Определение 5

Дуант – это полый металлический полуцилиндр, помещенный в вакуумную камеру между полюсами электромагнита в качестве одного из двух ускоряющих D-образного электрода в циклотроне.

К дуантам приложено переменное электрическое напряжение, чья частота эквивалентна циклотронной частоте. Частицы, несущие некоторый заряд, инжектируются в центре вакуумной камеры. В промежутке между дуантами они испытывают ускорение, вызываемое электрическим полем. Частицы, находящиеся внутри дуантов, в процессе движения по полуокружностям испытывают на себе действие силы Лоренца. Радиус полуокружностей возрастает с увеличением энергии частиц. Как и во всех других ускорителях, в циклотронах ускорение заряженной частицы достигается путем применения электрического поля, а ее удержание на траектории с помощью магнитного поля. Циклотроны дают возможность ускорять протоны до энергии, приближенной к 20 МэВ.

Однородные магнитные поля используются во многих устройствах самых разных типов назначений. В частности, они нашли свое применение так называемых масс-спектрометрах.

Определение 6

Масс-спектрометры – это такие устройства, использование которых позволяет нам измерять массы заряженных частиц, то есть ионов или ядер различных атомов.

Нужна помощь преподавателя?

Опиши задание — и наши эксперты тебе помогут!

Описать задание

Данные приборы используются для разделения изотопов (ядер атомов с одинаковым зарядом, но разными массами, к примеру, Ne20 и Ne22). На рис. 1.18.4 изображен простейшая версия масс-спектрометра. Вылетающие из источника S ионы проходят через несколько малых отверстий, которые в совокупности формируют узкий пучок. После этого они попадают в селектор скоростей, где частицы движутся в скрещенных однородных электрическом, создающимся между пластинами плоского конденсатора, и магнитном, возникающим в зазоре между полюсами электромагнита, полях. Начальная скорость υ→ заряженных частиц направлена перпендикулярно векторам E→ и B→.

Частица, которая движется в скрещенных магнитном и электрическом полях, испытывает на себе воздействия электрической силы qE→ и магнитной силы Лоренца. В условиях, когда выполняется E=υB, данные силы полностью компенсируют воздействие друг друга. В таком случае частица будет двигаться равномерно и прямолинейно и, пролетев через конденсатор, пройдет через отверстие в экране. При заданных значениях электрического и магнитного полей селектор выделит частицы, которые движутся со скоростью υ=EB.

После данных процессов частицы с одинаковыми значениями скорости попадают в однородное магнитное поле B→ камеры масс-спектрометра. Частицы под действием силы Лоренца движутся в камере перпендикулярной магнитному полю плоскости. Их траектории представляют собой окружности с радиусами R=mυqB’. В процессе измерения радиусов траекторий при известных значениях υ и B’, мы имеем возможность определить отношение qm. В случае изотопов, то есть при условии q1=q2, масс-спектрометр может разделить частицы с разными массами.

С помощью современных масс-спектрометров мы имеем возможность измерять массы заряженных частиц с точностью, превышающей 10–4.

Рисунок 1.18.4. Селектор скоростей и масс-спектрометр.

Магнитное поле

В случае, когда скорость частицы υ→ имеет составляющую υ∥→ вдоль направления магнитного поля, подобная частица в однородном магнитном поле будет совершать спиралевидное движение. Радиус такой спирали R зависит от модуля перпендикулярной магнитному полю составляющей υ┴ вектор υ→, а шаг спирали p – от модуля продольной составляющей υ∥ (рис. 1.18.5).

Рисунок 1.18.5. Движение заряженной частицы по спирали в однородном магнитном поле.

Исходя из этого, можно сказать, что траектория заряженной частицы в каком-то смысле «навивается» на линии магнитной индукции. Данное явление используется в технике для магнитной термоизоляции высокотемпературной плазмы — полностью ионизированного газа при температуре порядка 106 K. При изучении управляемых термоядерных реакций вещество в подобном состоянии получают в установках типа «Токамак». Плазма не должна касаться стенок камеры. Термоизоляция достигается путем создания магнитного поля специальной конфигурации. На рисунке 1.18.6 в качестве примера проиллюстрирована траектория движения несущей заряд частицы в магнитной «бутылке» (или ловушке).

Рисунок 1.18.6. Магнитная «бутылка». Заряженные частицы не выходят за ее пределы. Необходимое магнитное поле может быть создано с помощью двух круглых катушек с током.

Такое же явление происходит в магнитном поле Земли, которое защищает все живое от потока несущих заряд частиц из космического пространства.

Определение 7

Быстрые заряженные частицы из космоса, по большей степени от Солнца, «перехватываются» магнитным полем Земли, вследствие чего образуются радиационные пояса (рис. 1.18.7), в которых частицы, будто в магнитных ловушках, перемещаются туда и обратно по спиралеобразным траекториям между северным и южным магнитными полюсами за доли секунды.

Исключением являются полярные области, в которых часть частиц прорывается в верхние слои атмосферы, что может приводить к возникновению таких явлений, как «полярные сияния». Радиационные пояса Земли простираются от расстояний около 500 км до десятков радиусов нашей планеты. Стоит вспомнить, что южный магнитный полюс Земли находится поблизости с северным географическим полюсом на северо-западе Гренландии. Природа земного магнетизма до сих пор не изучена.

Рисунок 1.18.7. Радиационные пояса Земли. Быстрые заряженные частицы от Солнца, в основном электроны и протоны, попадают в магнитные ловушки радиационных поясов.

Возможно их вторжение в верхние слои атмосферы, служащее причиной возникновения «северных сияний».

Рисунок 1.18.8. Модель движения заряда в магнитном поле.

Рисунок 1.18.9. Модель Масс-спектрометра.

Рисунок 1.18.10. Модель селектора скоростей.

Сила лоренца. формула, определение

Пушка Гаусса

Математик Гаусс, когда познакомился с исследованиями Ампера, предложил создать оригинальную пушку (рис. 8), работающую на принципе действия магнитного поля на железный шарик – снаряд.

Рис. 8. Пушка Гаусса

Необходимо обратить внимание на то, в какую историческую эпоху были сделаны эти открытия. В первой половине XIX века Европа семимильными шагами шла по пути промышленной революции – это было благодатное время для научно-исследовательских открытий и быстрого внедрения их в практику

Ампер, несомненно, внес весомый вклад в этот процесс, дав цивилизации электромагниты, электродвигатели и телеграф, которые до сих пор находят широкое применение.

Следствия свойств силы Лоренца

Тело влетает в магнитном поле под каким-то определённым углом. Интуитивно понятно, что его величина имеет какое-то значение на характер воздействия на него поля, здесь нужно математическое выражение, чтобы стало понятнее. Следует знать, что как сила, так и скорость являются векторными величинами, то есть имеют направление. То же самое относится и к линиям магнитной напряженности. Тогда формулу можно записать следующим образом:

FЛ=qvBsinα,

sin α здесь – это угол между двумя векторными величинами: скоростью и потоком магнитного поля.

Как известно, синус нулевого угла также равен нулю. Получается, что если траектория движения частицы проходит вдоль силовых линий магнитного поля, то она никуда не отклоняется.

В однородном магнитном поле силовые линии имеют одинаковое и постоянное расстояние друг от друга. Теперь представим, что в таком поле перпендикулярно этим линиям движется частица. В этом случае сила Лоуренса заставит двигаться ее по окружности в плоскости, перпендикулярной силовым линиям. Чтобы найти радиус этой окружности, нужно знать массу частицы:

R=mvqB

Значение заряда не случайно взято как модуль

Это означает, что неважно, отрицательная или положительная частица входит в магнитное поле: радиус кривизны будет одинаков. Изменится только направление, в котором она полетит

Во всех остальных случаях, когда заряд имеет определенный угол α с магнитным полем, он будет двигаться по траектории, напоминающей спираль с постоянным радиусом R и шагом h. Его можно найти по формуле:

R=mvsinαqB

h=2mvcosαqB

Еще одним следствием свойств этого явления является тот факт, что она не совершает никакой работы. То есть она не отдает и не забирает энергию у частицы, а лишь меняет направление ее движения.

Самая яркая иллюстрация этого эффекта взаимодействия магнитного поля и заряженных частиц – это северное сияние. Магнитное поле, окружающее нашу планету, отклоняет заряженные частицы, прилетающие от Солнца. Но так как оно слабее всего на магнитных полюсах Земли, то туда проникают электрически заряженные частицы, вызывая свечение атмосферы.

Центростремительное ускорение, которое придается частицам, используется в электрических машинах – электродвигателях. Хотя уместнее здесь говорить о силе Ампера – частном проявлении силы Лоуренса, которая воздействует на проводник.

Принцип действия ускорителей элементарных частиц также основан на этом свойстве электромагнитного поля. Сверхпроводящие электромагниты отклоняют частицы от прямолинейного движения, заставляя их двигаться по кругу.

Самое любопытное заключается в том, что сила Лоренца не подчиняется третьему закону Ньютона, который гласит, что всякому действию есть свое противодействие. Связано это с тем, что Исаак Ньютон верил, что всякое взаимодействие на любом расстоянии происходит мгновенно, однако это не так. На самом деле оно происходит с помощью полей. К счастью, конфуза удалось избежать, так как физикам удалось переработать третий закон в закон сохранения импульса, который выполняется в том числе и для эффекта Лоуренса.

Использование


Эксперимент, показывающий воздействие силы Лоренца на заряженные частицы


Пучок электронов, движущихся по круговой траектории под воздействием магнитного поля. Свечение вызвано возбуждением атомов остаточного газа в баллоне

  • Основным применением силы Лоренца (точнее, её частного случая — силы Ампера) являются электрические машины (электродвигатели и генераторы). Сила Лоренца широко используется в электронных приборах для воздействия на заряженные частицы (электроны и иногда ионы), например в телевизионных электронно-лучевых трубках, а также в масс-спектрометрии и МГД-генераторах.
  • Сила Лоренца также используется в ускорителях заряженных частиц: она задаёт орбиту, по которой движутся эти частицы.
  • Сила Лоренца используется в рельсотроне.
  • Велосиметрия силой Лоренца заключается в бесконтактном измерении скорости движения проводящей жидкости.

Связь между силой Ампера и силой Лоренца

Действуя на проводник с током, магнитное поле воздействует на каждую заряженную частицу, создающую этот ток. А сила Ампера действует на весь проводник. Таким образом, сила Ампера равна сумме всех сил Лоренца, действующих на проводник с током.

FA= F·N

где F– сила Лоренца;

 N— число частиц.

Отсюда F= FAN

I = nqvS

N = nSl

Подставив эти выражения в формулу, получим выражение для силы Лоренца в магнитном поле:

F = qvBˑsinα.

Это выражение позволяет вычислить силу Лоренца в магнитном поле. Но магнитное поле не существует отдельно. Изменяясь, вместе с электрическим полем они порождают друг друга, образуя электромагнитное поле. А оно в каждой точке своего пространства характеризуется напряжённостью электрического поля Еи индукцией магнитного поляВ. И если электрически заряженная частица движется в электромагнитном поле, то на неё одновременно действуют и электрическое, и магнитное поле. Значит, величина силы Лоренца, действующая со стороны электромагнитного поля на частицу с зарядом q, движущуюся со скоростью v, зависит от этих величин:

F = q(E + vxB)

F, E, vиB) – векторные величины.

vxB– векторное произведение скорости движения частицы и индукции магнитного поля.

Направление силы Лоренца, как и силы Ампера, определяют с помощью правила левой руки: «Если расположить ладонь левой руки таким образом, чтобы линии магнитного поля входили в неё перпендикулярно, а 4 пальца направить в сторону движения частицы с положительным зарядом, или против движения частицы с отрицательным зарядом, то отогнутый на 90 большой палец покажет направление силы Лоренца».

Если заряженная частица движется параллельно силовым линиям магнитного поля, то величина силы Лоренца равна нулю, так как в этом случае α = 0, следовательно, sinα = 0

F = qvBˑsinα= 0.

Если же направление движения частицы перпендикулярно силовым линиям, то частица будет двигаться по окружности радиусом r, а сила Лоренца направлена к её центру, то есть является центростремительной силой.

Согласно второму закону Ньютона сила Лоренца равна mv2r.

Отсюда

При движении частицы под углом к силовым линиям её траектория представляет собой винтовую (спиральную) линию, имеющую радиус r и шаг винта h.

Сила Лоренца не совершает работы, так как её направление всегда перпендикулярно направлению движения заряда.

Немного истории

Первые попытки описать электромагнитную силу были сделаны еще в XVIII веке. Ученые Генри Кавендиш и Тобиас Майер высказали предположение, что сила на магнитных полюсах и электрически заряженных объектах подчиняется закону обратных квадратов. Однако экспериментальное доказательство этого факта не было полным и убедительным. Только в 1784 году Шарль Августин де Кулон при помощи своего торсионного баланса смог окончательно доказать это предположение.

В 1820 году физиком Эрстедом был открыт факт, что на магнитную стрелку компаса действует ток вольта, а Андре-Мари Ампер в этом же году смог разработать формулу угловой зависимости между двумя токовыми элементами. По сути, эти открытия стали фундаментом современной концепции электрических и магнитных полей. Сама же концепция получила свое дальнейшее развитие в теориях Майкла Фарадея, особенно в его представлении о силовых линиях. Лорд Кельвин и Джеймс Максвелл дополнили теории Фарадея подробным математическим описанием. В частности Максвеллом было создано так званное, «уравнение поля Максвелла» – представляющее собой систему дифференциальных и интегральных уравнений, описывающих электромагнитное поле и его связь с электрическими зарядами и токами в вакууме и сплошных средах.

Джей Джей Томпсон был первым физиком, кто попытался вывести из уравнения поля Максвелла электромагнитную силу, которые действует на движущийся заряженный объект. В 1881 году он опубликовал свою формулу F = q/2 v x B. Но из-за некоторых просчетов и неполного описания тока смещения она оказалась не совсем правильной.

И вот, наконец, в 1895 году голландский ученый Хендрик Лоренц вывел правильную формулу, которая используется и поныне, а также носит его имя, как и та сила, что действует на летящую частицу в магнитном поле, отныне называется «силой Лоренца».

Хендрик Лоренц.

Формула силы Лоренца при наличии магнитного и электрического полей

Если заряженная частица перемещается в пространстве, в котором находятся одновременно два поля (магнитное и электрическое), то сила, которая действует на нее, равна:

где – вектор напряженности электрического поля в точке, в которой находится заряд. Выражение (4) было эмпирически получено Лоренцем. Сила , которая входит в формулу (4) так же называется силой Лоренца (лоренцевой силой). Деление лоренцевой силы на составляющие: электрическую и магнитную относительно, так как связано с выбором инерциальной системы отсчета. Так, если система отсчета будет двигаться с такой же скоростью , как и заряд, то в такой системе сила Лоренца, действующая на частицу, будет равна нулю.

Сила Ампера

Существование силы Ампера подтверждает простой опыт. 

Если поместить между полюсами магнита проводник и пропустить по нему электрический ток, то можно увидеть, что проводник отклоняется от своего исходного положения. Это означает, что со стороны магнитного поля на него действует сила. Эта сила называется силой Ампера. Её величина определяется законом Ампера: «Со стороны магнитного поля на проводник с током действует сила, величина которой прямо пропорциональна силе тока, длине проводника в магнитном поле, модулю вектора магнитной индукции и синусу угла между вектором магнитной индукции и направление тока в проводнике». Математическое выражение этого закона выглядит так:

FA = I·l·В·sinα,

где I– величина тока в проводнике;

l– длина проводника с током в магнитном поле;

В – магнитная индукция;

α — угол между вектором магнитной индукции и направление тока в проводнике.

Примеры задачи

Задача 1

На заряд в 0,005 Кл, который движется в магнитном поле с индукцией 0,3 Тл, действует сила Лоренца. Вычислить ее, если скорость заряда 200 м/с, а движется он под углом 450 к линиям магнитной индукции.

Дано:

q = 0,005 Кл

B = 0,3 Тл

v = 200 м/с

α = 450

Решение:

В условиях задачи нет упоминания электрического поля, поэтому силу Лоренца можно найти по следующей формуле:

FЛ=qvBsinα=0,005×200×0,3×sin 450 =0,3×22=0,21 Н

Задача 2

Определить скорость тела, имеющего заряд и которое движется в магнитном поле с индукцией 2 Тл под углом 900. Величина, с которой поле воздействует на тело, равна 32 Н, заряд тела – 5 × 10-3 Кл.

Дано:

q = 0,005 Кл

B = 2 Тл

FЛ = 32 Н

α = 900

Решение:

Чтобы найти скорость заряда, необходимо несколько видоизменить формулу для нахождения силы Лоренца:

FЛ=qvBsinαv=FЛqBsinα

v=320,005×2×sin900=320,01×1=32000мс=32 км/с

Задача 3

Электрон движется в однородном магнитном поле под углом 900 ее силовым линиям. Величина, с которой поле воздействует на электрон, равна 5 × 10-13 Н. Величина магнитной индукции равна 0,05 Тл. Определить ускорение электрона.

Дано:

q = -1,6 × 10-19 Кл

B = 0,05 Тл

FЛ = 5 × 10-13 Н

α = 900

Решение:

В этой задаче сила Лоренца ко всему прочему еще и заставляет двигаться электрон по окружности. Поэтому здесь под ускорением следует понимать центростремительное ускорение:

aц=v2R

На данный момент неизвестны ни скорость электрона, ни радиус окружности, по которой он движется.

v=FЛqBsinα=5×10-13-1,6×10-19×0,05∙sin900=6×107мс

R=mvqB=9×10-31×6×107-1,6×10-19×0,05=6,8×10-3мс

aц=v2R=6×10726,8×10-3=5×1017мс2

Электродинамика оперирует такими понятиями, которым трудно подобрать аналогию в обычном мире. Но это совсем не значит, что их невозможно постичь. С помощью различных наглядных экспериментов и природных явлений процесс познания мира электричества может стать по настоящему захватывающим.

Определение и формула

Хендрик Лоренц доказал, что электромагнитная индукция взаимодействует с заряженными частицами. Эти взаимодействия приводят к возникновению силы Лоренца. Рассматриваемая сила возникает под действием магнитной индукции. Она перпендикулярна вектору скорости движущейся частицы (см. рис. 1). Необходимым условием возникновения этой силы является движение электрического заряда.

Рис. 1. Выводы Лоренца

Обратите внимание на расположение векторов (рисунок слева, вверху). Векторы, указывающие направления скорости и силы Лоренца, лежат в одной плоскости XOY, причём они расположены под углом 90º

Вектор магнитной индукции сориентирован вдоль оси Z, перпендикулярной плоскости XOY, а значит, в выбранной системе координат он перпендикулярен к векторам силы и скорости.

По закону Ампера:

Учитывая, что

(здесь j – плотность тока, q – единичный заряд, n – количество зарядов на бесконечно малую единицу длины проводника, S – сечение проводника, символом v обозначен модуль скорости движущейся частицы), запишем формулу Ампера в виде:

Так, как nSdl – общее число зарядов в объёме проводника, то для нахождения силы, действующей на точечный заряд, разделим выражение на количество частиц:

Модуль F вычисляется по формуле:

Из формулы следует:

  1. Сила Лоренца приобретает максимальное значение, если угол α прямой.
  2. Если точечный заряд, например, электрон, попадает в среду однородного магнитного поля, обладая некой начальной скоростью, перпендикулярной к линиям электромагнитной индукции, тогда вектор F будет перпендикулярен к вектору скорости. На точечный заряд будет действовать центробежная сила, которая заставит его вращаться по кругу. При этом работа равняется нулю (см. рис.2).
  3. Если угол между вектором индукции и скоростью частицы не равняется 90º, тогда заряд будет двигаться по спирали. Направление вращения зависит от полярности заряда (рис. 3).

Рис. 2. Заряженная частица между полюсами магнитовРис. 3. Ориентация вектора в зависимости от полярности заряда

Из рисунка 3 видно, что вектор F направлен в противоположную сторону, если знак заряда меняется на противоположный (при условии, что направления остальных векторов остаются неизменными).

Траекторию движения частицы правильно называть винтовой линией. Радиус этой винтовой линии (циклотронный радиус) определяется перпендикулярной к полю составной начальной скорости частицы. Шаг винтовой линии, вдоль которой перемещается частица, определяется составной начальной скорости заряда, вошедшего в однородное магнитное поле. Эта составная направлена параллельно к электромагнитным линиям.

В чём измеряется?

Размерность силы Лоренца в международной системе СИ – ньютон (Н). Разумеется, модуль силы Лоренца настолько крохотная величина, по сравнению с ньютоном, что её записывают в виде К×10-n Н, где 0

Когда возникает?

Магнитные поля не реагируют на неподвижный электрический заряд, так же как не действует сила Ампера на обесточенный проводник.

Для возникновения силы Лоренца необходимо выполнить три условия:

  1. У частицы должен быть отрицательный или положительный заряд.
  2. Заряженная частица должна находиться в магнитном поле.
  3. Частица должна быть в движении, то есть вектор v ≠ 0.

Если хотя бы одно из условий не выполняется, сила Лоренца не возникает.

Закон действия магнитного поля на проводник с током

Еще одно открытие Ампера – это закон действия магнитного поля на проводник с током. Он выражается прежде всего в действии магнитного поля на виток или рамку с током. Так, на виток с током в магнитном поле действует момент силы, которая стремится развернуть этот виток таким образом, чтобы его плоскость стала перпендикулярна линиям магнитного поля. Угол поворота витка прямо пропорционален величине тока в витке. Если внешнее магнитное поле в витке постоянно, то значение модуля магнитной индукции также величина постоянная. Площадь витка при не очень больших токах также можно считать постоянной, следовательно, справедливо то, что сила тока равна произведению момента сил, разворачивающих виток с током, на некоторую постоянную при неизменных условиях величину.

 – сила тока,

 – момент сил, разворачивающих виток с током.

Следовательно, появляется возможность измерять силу тока по величине угла поворота рамки, которая реализована в измерительном приборе – амперметре (рис. 2).

Рис. 2. Амперметр

КП.Сила Ампера.Сила Лоренца.10клdocx

 Изучение нового материала. 

Действия учителя:Дать возможность самостоятельно определить значение физических явлений в природе.Основная учебная проблема при рассмотрении нового материала — изучение характеристик, которые можно использовать для описания свойств магнитного поля.Необходимо повторить и углубить представление о векторе магнитной индукции В. 

1)      Чтохарактеризует величину В?( Это векторная характеристика магнитного поля: она имеет направление и числовое значение).

2)      Что называют линией магнитной индукции?

3)      Для чего она вводится?

4)    Есть ли линии магнитной индукции в природе?

Действия учащихся. Далее изучают магнитные поля по картине линий магнитной индукции, отрабатывают правило буравчика. Примеры заданий (рисунки выполнены на слайде).

1. Известно направление линий магнитной индукции (рис. 3, 4). Укажите направление тока в проводнике.

Рис 1 Рис. 2 Рис. 3

2.    2. По расположению магнитных стрелок определите направление тока в проводнике (рис.).

3.    3.Как установится магнитная стрелка, если по проводнику пропустить постоянный электрический ток (рис.)?

4.    4.Определите полюсы источника питания, если магнитная стрелка около проводника ориентирована так, как показано на рисинке.

Как определить направление вектора магнитной индукции В — основная учебная проблема урока.

Демонстрация. Демонстрируется действие магнитного поля подковообразного магнита на проводник с током (элемент тока). 

Вопросы для организации беседы.

1)      Зависит ли отклонение провод-пика с током (сила, действующая на проводник) от силы тока?

2)      Зависит ли отклонение проводника с током от длины проводника?

3)      Зависит ли характеристика В магнитного поля от силы тока, от длины проводника? (Ответ. Нет, не зависит.)

Записывают в тетради по учебнику: Вектор магнитной индукции можно принять за характеристику магнитного поля, так как оно не зависит пи от силы тока, ни от длины проводника.

Вопрос.В чем смысл закона Ампера?

Нам известен экспериментальный факт: магнитное поле действует на проводник с током.В 1826 г. французский физик А. Ампер сформулировал закон,описывающий это действие магнитного поля. Закон представленвыражением F = В \ I \ l sin а. При этом направление силы определяется по правилу левой руки.

При объяснении материала важно сравнительно быстро ввести закон, а усвоение отрабатывать при решении задач.

У любой физической величины есть единица.

Учитель. Дает определение единицы индукции В — тесла.

Если левую руку расположитьтак, чтобы вектор магнитной

индукции входил в ладонь, авытянутые пальцы былинаправлены вдоль тока, тоотведенный большой палецукажет направление действиясилы Ампера на проводник с током.

Вывод.

1.Сила Ампера – сила, действующая на проводник тока, находящийся в магнитном поле и равная произведению силы тока в проводнике, модуля вектора индукции магнитного поля, длины проводника и синуса угла между вектором магнитного поля и направлением тока в проводнике.

2.Сила Ампера принимает своё наибольшее значение когда векторы индукции и направления тока перпендикулярны.

3.Если вектор магнитной индукции входит в ладонь левой руки и четыре пальца вытянуты в сторону направления вектора движения тока, тогда отогнутый в сторону большой палец показывает направление силы Ампера.

Учитель.Т.к. магнитное поле действует на ток – движущиеся заряженные частицы, то оно действует и на каждую частицу в отдельности. Действие магнитного поля на движущуюся заряженную частицу характеризует сила Лоренца.

Самостоятельное изучение темы: построение новых знаний на базе предыдущих.

Критерии оценивания.

·         Умение искать и выделять необходимую информацию.

·         Умение оперировать гипотезами.

·         Умение аргументировать свою точку зрения.

Ученик1.Хендрик Антон Лоренц (1853–1928) выдающийся голландский физик и математик , развил электромагнитную теорию света и электронную теорию материи, а также сформулировал теорию электричества, магнетизма и света, внёс большой вклад в развитие теории относительности, лауреат Нобелевской премии 1902г.

, так как является её долей, значит, для определения её направления можно применить то же мнемоническое правило, что и для определения направления сил Ампера – правило левой руки, с оговоркой, что заряд должен быть положительным, т.к. за направление тока мы принимаем направление движения положительных зарядов. Если же заряд отрицательный, то направление силы меняется на противоположное.Так как сила, действующая на заряд, оказалась перпендикулярной скорости его движения, то модуль скорости изменяться не будет, а будет меняться направление, т.о. частица будет равномерно двигаться по окружности.

Учитель.

       I.            Выведем формулу для расчёта модуля силы Лоренца.Формулу силы Лоренца можно найти с помощью силы Ампера.Так как Fл= F/N” где F- сила Ампера, N – число заряженных частиц

I = q n v S – сила тока; F= |I| ΔL В    ; Подставим выражение для силы тока и получим:

F=|q| n v S ΔL Bsinα = v |q| N Вsinα; где N = n S ΔL- число заряженных частиц в единице объема.

Значит , на каждую движущуюся заряженную частицу магнитное поле действует с силой Лоренца:

Fл= F/N = |q| v B , где α- угол между вектором скорости и вектором

     II.            Направление силы Лоренца определяют с помощью правила левой руки.

Ладонь левой руки располагают так, чтобы составляющая магнитной индукции входила в ладонь, четыре, вытянутые пальца, показывали направление движения положительного заряда

(против движения отрицательного), то тогда отогнутый на 90˚большой палец укажет направление действующей на заряд силы Лоренца 

На заряженную частицу одновременно действуют магнитное и электрическое поля, значит полная сила будет равна: F̄= F̄ЭЛ+F̄Л Под действием силы Лоренца не меняется модуль ее скорости, а меняется только направление скорости частицы.

Демонстрация наблюдения силы Лоренца с помощью осциллографа и постоянного магнита.

http://home-task.com/plan-konspekt-uroka-po-fizike-tema-dejstvie-magnitnogo-polya-na-dvizhushhijsya-zaryad-sila-lorenca/

Движение заряженной частицы под действием силы Лоренца, если α = 90°

Сила, перпендикулярная скорости, вызывает изменение направления движения, т.е. центростремительное ускорение. Зная формулы расчёта центростремительного ускорения и модуля силы Лоренца, которая его вызывает, и, используя второй закон Ньютона, выведите формулу для расчёта радиуса окружности, по которой будет двигаться частица.

Теперь не сложно узнать и период обращения частицы, т.к.  , где r нами только что найдено.

В.Сделайте вывод: чем определяется период обращения частицы?(Предполагаемый ответ: магнитной индукцией поля и удельным зарядом частицы, т.е. не зависит от радиуса окружности, по которой частица движется.)

  III.            Демонстрация наблюдения силы Лоренца с помощью осциллографа и постоянного магнита.

  IV.            Применение силы Лоренца.

Закрепление изученного материала

№1.Определить, используя рисунок направления В, FЛ, V; применяя правило левой руки.

№ 2.Электрон под действием однородного магнитного поля обращается по окружности радиуса R с периодом Т. Какими станут значения радиуса окружности и периода обращения электрона при увеличении индукции магнитного поля в 2 раза?

http://home-task.com/plan-konspekt-uroka-po-fizike-tema-dejstvie-magnitnogo-polya-na-dvizhushhijsya-zaryad-sila-lorenca/

     V.            Самостоятельная работа.

(с целью закрепления полученных знаний, в ходе которой они могут пользоваться своими записями, текстом учебника, консультацией учителя.)

1.Проверка основных формул (дописать пропущенные физические величины)

В= F/I…; Fл= eB…sinα; A= …U; B= μ0μN…/ℓ; T=2π…/υ; E=F/…; Fa=B……sinα; F=mац=m…/r.

2.Проверка единиц измерения физических величин:

Тл= Н/А*м; В= Дж/Кл; Гн= В*с/А; Н= кг* м/с2; Кл=А*с; Дж=Н*м; Вб=Тл*м2

Εi F I Ф

q L A B

3. Проверка правила левой руки.

4.Самостоятельное решение задач. Парная работа.

№ 1. Циклотрон предназначен для ускорения протонов до энергии 5 МэВ. Определить наибольший радиус орбиты, по которой движется протон, если индукция магнитного поля 1Тл.

№ 2. В направлении, перпендикулярном линиям магнитной индукции влетает электрон, скорость которого 10 Мм/с. Электрон описал в магнитном поле окружность радиусом 1 см. Чему равна индукция этого магнитного поля?

№ 3.Электрон движется в однородном магнитном поле с индукцией 4 мТл. Чему равен период обращения электрона?

№ 4. Можно ли неизолированный провод намотав, на железный сердечник получить самодельный электромагнит? ( Ответ: нет)

№ 5. Почему магнитное действие катушки, по которой идет ток, усиливается, если в нее ввести железный сердечник?

№ 6. Почему корпус компаса делают из меди, алюминия, пластмассы и других материалов, но никогда не делают его из железа?

№ 7. Если магнит дугообразный, то гвоздь одним концом притягивается к одному полюсу, а другим – к другому. Почему?

http://home-task.com/reshenie-zadach-po-teme-sila-lorenca/

электромагнетизм — есть ли интуитивное объяснение того, почему сила Лоренца перпендикулярна скорости частицы и магнитному полю?

Псевдовектор аргумент

Существует интуитивно понятный аргумент, но первое, что нужно сделать, — это взять двойственную по Пуанкаре к B. В трех измерениях существует эпсилон-тензор $ \ epsilon_ {ijk} $, который является инвариантным — он не меняется под действием вращения. Он имеет $ \ epsilon_ {123} = 1 $, и все перестановки дают знак минус, так что значение $ \ epsilon $ равно нулю из двух индексов, равных, и знак перестановки, чтобы получить 123, если они все разные.{\ sigma} $, тензор эпсилона, стянутый с B. Это антисимметричный тензор ранга 2. Поскольку тензор $ \ epsilon $ инвариантен, антисимметричный тензор эквивалентен вектору при поворотах, но не эквивалентен при отражениях. Причина в том, что вектор меняет знак при отражении, а тензор — нет. Это также верно для B — это псевдовектор, если вы отражаете пространство с помощью токоведущего провода, направление B не меняется на противоположное.

Тот факт, что B по сути является тензором, а не вектором, означает, что, когда он взаимодействует с частицей со скоростью v, он может образовывать силу только тогда, когда один из индексов сжимается с чем-то.\ mu $ как сила, а это $ v \ times B $

В теории относительности это кажется единственной естественной вещью, поскольку поля E и B вместе образуют антисимметричный 2-тензор, а 4-сила Лоренца — это тензор, сжатый с 4-скоростью. Эта форма настолько естественна и интуитивно понятна, что не требует подробного обоснования.

Более физическое повторение аргумента

Вышесказанное является своего рода формальным звучанием, но оно просто говорит следующее: магнитное поле не меняет знак при изменении координат пространства на противоположное.Чтобы увидеть это физически, рассмотрим соленоид тока, растянутый вдоль оси z от -a до a, с током в основном в плоскости x-y вдоль каждой обмотки, и отразите этот соленоид по осям x-y-z. Отражение x меняет направление тока, отражение y возвращает его туда, откуда он начался, а отражение z не меняет соленоид.

Так как ток такой же, то В то же самое! Таким образом, B от соленоида не изменяется при отражении. Таким образом, сила, действующая на частицу, не может быть направлена ​​в направлении B, потому что сила меняет направление на противоположное при отражении, а B — нет.Сила может быть только в направлении величины, которая меняет направление, и простейшая такая величина — $ v \ times B $. При отражении v меняет направление на противоположное, а B — нет, поэтому сила Лоренца полностью меняет направление.

Этот аргумент предполагает симметрию отражения, которая является симметрией электромагнетизма, но на самом деле не является фундаментальной симметрией в нашей Вселенной. Тот же аргумент отражения показывает, что магнитный заряд не является должным образом симметричным с электрическим зарядом, так как магнитный заряд меняет знак при отражении (отразите все координаты с зарядом в начале координат — поле перемещается в новое место, но указывает на то же самое. направление, поэтому смысл магнитного заряда меняется на противоположное).Это свойство означает, что магнитные монополи были ранним признаком того, что природа не инвариантна по четности, и может объяснить, почему Дирак не удивился, когда было показано, что слабые взаимодействия нарушают четность.

Другое предположение состоит в том, что сила — это простейшая инвариантная к отражению комбинация B и v. Если вы откажетесь от идеи, что сила линейно пропорциональна B, есть более сложные комбинации, которые также работают, чтобы дать силовой закон, инвариантный к отражению. Эти комбинации обычно не соблюдают закон сохранения энергии.

Чтобы иметь автоматическое сохранение энергии (и автоматическое фазовое пространство с симплектическими свойствами), вы должны вывести свои уравнения движения из действия. 2 \ over 2m $, а потенциальная энергия равна $ e \ phi $.2 \ более 2м + \ phi} $$ $$ \ partial_t x = {p — eA \ over m} $$

И объединение уравнений в уравнение второго порядка для ускорения x дает закон силы Лоренца. Такая же замена в гамильтониане, $ p $ на $ p-eA $, работает в теории относительности и дает правильный четырехмерный закон силы Лоренца.

Отождествление B с $ \ nabla \ times A $ может быть оправдано инвариантностью уравнений при добавлении градиента к A. Классической физической частью A является его локон, и это разумно отождествлять с B в уравнении Максвелла. уравнения.

Этот аргумент принципиально верен, потому что он не зависит от инвариантности отражения (любой аргумент, основанный на симметрии отражения, на самом деле фальшивый, поскольку мы знаем, что это не симметрия природы ни в каком фундаментальном смысле), и верен квантово-механически, когда вы интерпретировать калибровочную инвариантность как свободу в переопределении локальной фазы волновой функции заряженной частицы. Единственный недостаток состоит в том, что для этого требуется некоторое знакомство с принципом Гамильтона.

Добро пожаловать в Donald A.Glaser Physics 111 B Экспериментальная лаборатория

ЭТОТ ВЕБ-САЙТ НЕ ИСПОЛЬЗУЕТСЯ ОСЕНЬЮ 2020 И ВЕСНОЙ 2021. ПОЖАЛУЙСТА, ПРОСМОТРЕТЬ bCourses ВМЕСТО

Профессор Дональд А. Глейзер на протяжении всей своей карьеры был мастером экспериментальной науки. Он родился в Кливленде, получил образование в Технологическом институте Кейса, получил докторскую степень в Калифорнийском технологическом институте и преподавал в Мичиганском университете, прежде чем занять должность в Калифорнийском университете в Беркли в 1959 году. В начале своей карьеры доктор Глейзер экспериментировал с способами заставить работать принцип видны субатомные частицы.За последующее изобретение пузырьковой камеры он был удостоен Нобелевской премии по физике 1960 года. Затем он начал исследовать новую область молекулярной биологии, совершенствуя методы работы с бактериальными фагами, бактериями и клетками млекопитающих. Разработав оборудование для автоматизации своих экспериментов и их масштабирования, он мог проводить тысячи экспериментов одновременно, генерируя достаточно данных, чтобы продвинуть науку вперед. Осознавая последствия для медицины, доктор Глейзер и двое его друзей создали в 1971 году новаторскую биотехнологическую компанию Cetus Corporation, тем самым положив начало индустрии генной инженерии.

Позже в своей карьере доктор Глейзер продолжил свои экспериментальные исследования, разрабатывая математические модели для понимания фундаментальных процессов человеческого зрения. На протяжении всей своей жизни он сочетал научную работу с не менее страстным интересом к классической музыке, везде играл на альте в ансамблях камерной музыки. Жадно любопытный, новаторский и преданный развитию научных знаний, доктор Глейзер был очень востребованным консультантом и советником до своей смерти в 2013 году.

Экспериментальная лаборатория

Physics 111 — это интенсивный лабораторный курс из трех частей для студентов 3-го и 4-го курсов физики Калифорнийского университета в Беркли. Это следует за Physics 111A Instrumentation Laboratory , которая знакомит студентов с электроникой, методами измерения, обработкой сигналов, компьютеризированным сбором данных и управлением. С помощью этих инструментов учащиеся секции «Экспериментальная лаборатория» курса «Физика 111» проводят четыре эксперимента, на выполнение каждого из которых уходит около 2 недель.Студенты выбирают эти эксперименты из 19 или около того экспериментов, постоянно установленных в 111-Lab. Они представляют широкий спектр тем и методов, используемых в экспериментальной физике. Многие эксперименты повторяют исследования, получившие Нобелевскую премию, и все они предназначены для развития навыков, необходимых для исследования.


Что делать в первую очередь Перед тем, как прийти в лабораторию физики 111-Lab

  • Чтобы уменьшить накладные расходы и риск получения несогласованных данных, вы должны убедиться, что у вас есть блоки последовательных дней для завершения некоторых экспериментов (см. Список доступных экспериментов для этого требования Подписки оборудования).
  • Все студенты входят в bCourses, заполняют карточку подписи и отправляют ее через Интернет, прежде чем вы придете в лабораторию. Вы должны выполнить это, прежде чем начинать что-либо в лаборатории.
  • Все студенты приходят на ознакомительную встречу в 286 LeConte Hall в первый учебный день в 13:00.
  • Каждый студент должен иметь свою фотографию на сайте bCourses Калифорнийского университета в Беркли. Если НЕТ, вы должны сдать ФОТОГРАФИЮ НА ПАСПОРТ (не свой паспорт, а настоящую фотографию паспортного типа) в первый день Лаборатории в целях безопасности.
  • Все студенты экспериментальной лаборатории по физике 111B должны выполнить упражнение по анализу ошибок , назначенное в течение первой недели (см. Сроки выполнения).
  • Приготовьтесь к одному обязательному устному докладу с профессором. Это может быть первый, второй или третий эксперимент.
  • При доступе к компьютерам 111B-Lab, если вы НЕ можете войти в систему, сделайте следующее:
    Перейдите в Calnet Active Directory и синхронизируйте свое имя CalNet.

Внимание: в 111-Lab НИКОГДА нельзя есть и пить, кроме как на скамейках с СИНЕЙ полосой около в комнатах 282 и 286 LeConte .

В начало

В начало

В начало

Об этом сайте

Этот сайт дополняет сайт «Инструменты» Physics 111A Instrumentation Course, предоставляя лабораторные руководства для каждого эксперимента в Physics 111B Experimentation Лаборатория . Каждая ссылка в меню «Эксперименты в экспериментальной лаборатории» вверху ведет на страницы, содержащие руководство по эксперименту, которое включает в себя предварительные вопросы, промежуточные вопросы, справочные материалы, теорию и инструкции.

В начало

Для студентов, в настоящее время обучающихся на этом курсе

Вы можете просматривать и распечатывать любые страницы на сайте экспериментальной лаборатории без учетной записи или входа в систему.

Если вы видите ошибки в описаниях лабораторных работ или места, которые требуют улучшения, поговорите со своими инструкторами или Winthrop Williams . Они могут быстро редактировать страницы, внося незначительные изменения, чтобы следующий ученик получил улучшенную версию. Если вы видите необходимость внесения значительных изменений, распечатайте страницу и напишите свои предложения, чтобы передать их инструктору.Вы также можете оставить комментарии в оценочном опросе внизу описания, и мы внесем изменения по мере необходимости, если они предложат некоторые полезные улучшения. Мы действительно ценим вклад студентов в улучшение лабораторных работ.

В начало

Для других посетителей

Этот сайт открыт для всех, но не для редактирования и комментирования. Авторские права принадлежат Регентам Калифорнийского университета. Однако мы с радостью предоставляем контент другим школам для некоммерческого использования в образовательных целях.Некоторые ссылки на защищенные авторским правом ссылки и программное обеспечение недоступны никому без аутентификации в качестве студента или сотрудника Physics 111.

В начало

В начало

Благодарности

Этот сайт был разработан Доном Орландо и Сегре Студентами. Стажер Segrè, Натан Белмор, провел большую часть июля 2014 года, преобразовывая лабораторные описания из Wiki в Drupal, проектируя новый веб-сайт и преобразовывая все коды.Уинтроп Уильямс в настоящее время администрирует веб-страницы. Преподавательский состав и Уинтроп Уильямс занимаются редактированием и обслуживанием.

Персонал лаборатории:

В начало

Домашняя страница физического факультета

ПОЧЕМУ МАГНИТНЫЕ СИЛЫ ЗАВИСЯТ ОТ КТО ИХ ИЗМЕРЕНИЯ

ПОЧЕМУ МАГНИТНЫЕ СИЛЫ ЗАВИСЯТ ОТ КТО ИХ ИЗМЕРЕНИЯ ПОЧЕМУ ЗАВИСЯТ ЛИ МАГНИТНЫЕ СИЛЫ ОТ КТО ИХ МЕРЯЕТ?

Дэвид Н. Джеймисон PhD.
Школа физики
Мельбурнский университет

Введение: Магнетизм и силы природы

В марте 1989 года мощные магнитные бури на Солнце, вызванные необычным активность солнечных пятен вызвала отключение электроэнергии в Квебеке, Канада, что повлияло на более шести миллионов человек.

Машинист поезда Met замыкает выключатель, вызывая протекание электрического тока на две катушки из проволоки, одна из которых прочно прикреплена к шасси поезда, другой закреплен на колесах. Мощные магнитные скручивающие силы приводят в движение несколько сотен тонн стали и людей на линии.

На Мельбурнском ядерном микрозонде, специальном электромагните, который генерирует сложная картина магнитных полей, вызывает пучок заряженных частиц сходиться к крошечной точке диаметром в одну миллионную метра.Это используется как мощный диагностический зонд в медицине, ботанике, геологии и материалах наука.

Бушуокер, оснащенный магнитным компасом, умеет уверенно перемещаться по безлюдной пустыне, следуя магнитным полям Земли. поле.

Сверхчувствительный датчик магнитного поля, называемый сверхпроводящим квантом. Устройство помех, настраивается на слабые магнитные поля, создаваемые мысли, бегающие по нервным путям в человеческом мозгу.

Рисунок 1: Сильные магнитные поля в солнечных пятнах вызывают расщепление солнечного света. Мы можем воспроизвести тот же эффект в лаборатории. с помощью мощных магнитов. (Из Т. Хей и П. Уолтерс, «Квантовая Вселенная», Издательство Кембриджского университета, Кембридж, 1988 г.)

Магнитные силы повсеместно распространены в мире природы и имеют значительную диапазон приложений в нашей технологической цивилизации. Но есть что-то беспокоясь о магнетизме.Будучи студентом, изучающим физику, я был беспокоился, что магнитные силы ощущаются только движущимися заряженными частицами. Это видно из основной формулы для силы, F , на частице с зарядом q , движущейся со скоростью v через область магнитного и электрического полей:

F = q E + q v ´ В

, где E и B — сильные стороны электрического и магнитные поля.Эта сила называется силой Лоренца. Сила от электрического поля (бит q E ) кажется прямым достаточно, но определенно величина магнитной силы ( q v ´ B бит) зависит от того, кто его измеряет? Это потому, что мои измерения скорость частицы будет зависеть от того, насколько быстро я иду относительно частица! То есть скорость моей личной системы отсчета относительно частицы.Конечно, выбор системы отсчета не должен повлиять на магнитную силу? В самом деле, что происходит с магнитной силой в система отсчета самой частицы, где скорость равна нулю? Магнитная сила тоже равна нулю? Если этого кажущегося парадокса недостаточно, многие формулы магнетизма удивительно похожи на эквивалентные формулы, которые применяются к электростатическим силам, но имеют некоторые предполагаемые асимметрии. Подробнее об этом позже.

Цель этого эссе — выделить некоторые моменты, которые магнетизм отдельно от других сил, затем объясните, почему это должно быть так.

Рисунок 2: Четыре силы природы.

Давайте рассмотрим различные типы силы и посмотрим, где подходит магнетизм. дюйма. В природе есть четыре силы (недавние предположения, что быть «пятой силой» экспериментально не доказано). Это, начиная с с наиболее знакомыми:

  • Сила тяжести. Это сила, которая поднимает и опускает нас, большинство из нас (кроме от нескольких астро- и космонавтов) провести всю свою жизнь, погрузившись в гравитационное поле Земли.
  • Электромагнетизм. Это объединяет электростатические и магнитные силы вместе, по причинам, которые, я надеюсь, станут понятны в конце этой лекции. Большинство люди будут знакомы с электростатическими силами с точки зрения из пластика (в древности кусок янтаря), при натирании кусочком ткани, подберет небольшие кусочки бумаги. Электростатические силы также не дай нам провалиться в пол! Это отталкивание между поверхностью электроны, которые мешают двум объектам занимать одно и то же место.Некоторый уже были описаны знакомые примеры магнитной силы.
  • Сильная ядерная сила. Это сила, которая связывает нейтроны и протоны. вместе в ядре атомов. Он действует только на небольшом расстоянии, но значительно более мощная, чем электростатическая сила, которая в противном случае заставляют положительно заряженные протоны сильно отталкиваться друг от друга. В средний человек не испытывает эту силу напрямую, но я вижу достаточно доказательства этого, когда я бомбардирую легкие элементы, такие как углерод, быстрыми протонами в рамках моей исследовательской деятельности.Взаимодействие протона с на ядро ​​углерода сильно влияет сильная сила.
  • Слабая ядерная сила. Эта сила, среди прочего, участвует в некоторые виды радиоактивного распада. Наши собственные тела содержат легко измеримые количество радиоактивного изотопа калия, который тихо распадается прочь из-за действия слабого ядерного взаимодействия.
Список сил указывает на то, что магнетизм тесно связан с электростатических сил, так как он не фигурирует в списке как отдельный сила.Давайте теперь посмотрим на историю магнетизма, чтобы найти еще несколько ключей.

История магнетизма

ок. 1000 г. до н. Э .: Согласно классическому греческому историку Плинию, слово магнетизм происходит от имени мальчика-пастуха по имени Магнес, который находит что его посох с железным наконечником привлекают глыбы естественного происхождения. магнетит (магнитный оксид железа) на горе Ида, Греция. Примерно в то же время Китайские мореплаватели обнаруживают, что кусок магнита (другое название магнетит), подвешенный на нити, всегда указывает в одном направлении.В история о мальчике-пастухе скорее легендарная, чем историческая, но в течение следующих 3000 лет изучение магнетизма естественным образом ограничится возникающие постоянные магниты.

Рисунок 3: Открытие магнетизма. (Из Л. ДеВриса, «The Книга экспериментов «, Джон Мюррей, Лондон, 1958 г.)

1269: Пьер Пелерен де Марикур обнаруживает, что сферический У магнитного камня есть два места на поверхности, где сходятся магнитные силовые линии.Он называет их «полюсами» по аналогии с полюсами Земли.

Исследование постоянных магнитов природного происхождения обнаруживает еще несколько важные свойства. Выяснилось, что разрушение магнита пополам всегда в результате получается два новых магнита, каждый со своим северным и южным полюсами. Это никогда невозможно создать магнит, который имеет только север или только юг. столб. Это совсем не похоже на поведение заряженных объектов, где они легко можно дать объекту либо положительный, либо отрицательный заряд.Постоянные магниты всегда выглядят как диполи . Это еще одна подсказка что в магнетизме есть что-то необычное.

1600: Чтобы объяснить, как работает компас, Уильям Гилберт (1540–1603) постулирует, что Земля действует как огромная сферическая магнит с северным и южным полюсами. Гилберту также приписывают первым использовал термины «электрическая сила» и «электричество», которые производят от греческого слова «янтарь».

1820: При организации демонстрации лекций для класса физики а Датский преподаватель физики Ханс Кристиан Эрстед (1777-1851), замечает, что электрический ток в проводе отклоняет стрелку компаса.В электрический ток создавал магнитное поле! Наука «электромагнетизм» основан.

1821: Андре Мари Ампер (1775-1836) определяет закон, который теперь носит его имя, для отношений между ток, протекающий по проводу, и создаваемое им магнитное поле.

1831: Майкл Фарадей (1791-1867) и, независимо, Джозеф Генри (1797-1878) показывает, как меняющиеся магнитные поле, пронизывающее проволочную петлю, может вызвать электрический ток в петле.Фарадей опубликовал первым, поэтому теория результата теперь называется «Закон индукции Фарадея». Большая часть электроэнергии вырабатывается за счет эксплуатации Закон Фарадея.

1873: Джеймс Клерк Максвелл (1831-1879) издает его «Трактат об электричестве и магнетизме», содержащий исчерпывающий теория всех открытий до сих пор. Однако это одинокий британский чудак. Оливер Хевисайд (1850-1925), который дистиллирует суть теории из чрезвычайно сложной математики Максвелла в элегантно простая форма, известная сегодня как уравнения Максвелла.

1895: Хендрик Лоренц (1853-1944) решает что уравнения Максвелла можно понять на фундаментальном уровне как взаимодействие движущихся заряженных частиц. Только в 1899 году эти движущиеся заряженные частицы становятся известными как электрон . Лоренц вводит формулу для силы, действующей на заряженную частицу движущиеся в электрическом и магнитном полях, которые мы теперь называем силой Лоренца, обсуждалось ранее.

Рисунок Рисунок 4: Уравнения Максвелла.

Хотя уравнения Максвелла удивительно симметричны между роли электрического и магнитного полей, есть некоторые вопиющие асимметрии. Эти асимметрии связаны с отсутствием магнитного эквивалента электрона: магнитный монополь. Например закон Гаусса для магнетизма отрицает роль магнитных монополей в происхождении магнитного поле. Это согласуется с экспериментом, поскольку магнитные монополи ни разу не были найдены, несмотря на тщательный поиск.Также закон Фарадея не содержит термина «монопольные токи», который был бы аналогичен термин для электрического тока в законе Ампера-Максвелла.

Уравнения Максвелла говорят нам, как генерируются магнитные поля, либо электрическими токами или изменяющимися электрическими полями. Формула силы Лоренца говорит нам, как магнитные поля влияют на движущиеся заряженные частицы. Но как это то, что постоянные магниты могут генерировать магнитные поля, по-видимому, без электрические токи, и почувствовать действие магнитных сил, по-видимому, без содержащие движущиеся заряженные частицы? Ответ в том, что они действительно содержат движущиеся заряженные частицы! Еще один результат конца девятнадцатого век нужен:

1896: Питер Зееман (1865-1943) открывает эффект, за который он разделит Нобелевскую премию 1902 года с Лоренцем, что спектральные линии будут расширяться, если источник находится в магнитном поле.Эксперименты 1897 года показывают фактическое расщепление линий. Эти результаты интерпретируется как действие силы Лоренца на движущиеся электроны внутри Атомы . Теперь мы знаем, что электроны в атомах прыгают между уровнями энергии. образуют спектральные линии, поэтому любое влияние на их движение проявляется в сдвиг строк.

Сегодня мы рассматриваем вращающиеся электроны внутри атомов как квантовые механические стоячие волны. Несмотря на это, электроны по-прежнему действуют как крошечные петли тока, бегущие вокруг атома.Уравнения Максвелла говорят нам что эта петля тока будет генерировать магнитное поле. В большинстве атомов магнитные поля, создаваемые всеми этими крошечными токовыми петлями, уравновешиваются. Но в железе и некоторых других ферромагнитных материалах этого не происходит, каждый атом действует как крошечный магнит. При благоприятных обстоятельствах многие из них крошечные атомные магниты заблокированы в выравнивании, и, следовательно, у нас есть постоянный магнит. Выровненные атомные магниты затем могут вытягиваться и временно выравниваться крошечные магниты в кусках немагнитного железа и притягивают их.Также, они могут притягивать или отталкивать другие магниты в зависимости от того, противоположны они или одинаковые полюса вместе. Эффект совмещения всех выровненных прядений Электроны в ферромагнитном материале называют «током решетки».

Рисунок 5: (слева) линия гамма-излучения из первого возбужденного состояния. из 57 Fe в немагнитной нержавеющей стали. (справа) тот же гамма-луч, на этот раз от 57 Fe, внедренного в магнитный ком железа.Сильные внутренние магнитные поля от решеточных токов в железе вызывают зеемановское расщепление. (Из студенческого эксперимента по эффекту Мёссбауэра, Школа физики Мельбурнского университета, 1985)

Итак, мы видим, что как в постоянных магнитах, так и в электромагнитах движение заряженных частиц, обычно это токи электронов, необходим для генерации магнитные силы, и, в свою очередь, движение заряженных частиц имеет важное значение «почувствовать» магнитные силы.

В пространстве было множество беспокойно движущихся заряженных частиц. примеры магнетизма в начале этого эссе. Токи термоядерной плазмы вокруг солнечных пятен на Солнце создают магнитные поля, которые затем нарушают токи в канадской электросети. Электрические токи в обмотки возбуждения двигателей поездов создают магнитные поля, которые действуют на токи в обмотках якоря, заставляющие их вращаться. Токи в обмотках специальных электромагнитов на Melbourne Nuclear Микрозонд создает магнитные поля, которые заставляют широкую заряженную частицу Луч сходится к тонкому зонду.В Земле «магнитогидродинамические» токи в ядре генерируют магнитное поле Земли (с помощью механизма, который еще не полностью изучен), который затем действует на токи решетки в намагниченная стрелка компаса заставляет его указывать на север. В человеческом мозгу слабые электрические токи, проходящие по нервам, создают магнитные поля которые можно обнаружить по тонким эффектам на своеобразные сверхтоки электронов. пары в сверхпроводнике.

Итак, наконец в нашей истории магнетизма мы подошли к золотому году для физика:

1905: Альберт Эйнштейн (1879-1955) издает его статья «Электродинамика движущихся тел», содержащая специальный Теория относительности.Позднее Эйнштейн заметил об этой статье:

«Что привело меня более или менее прямо к специальной теории относительности было убеждение, что электродвижущая сила, действующая на движущееся тело в магнитном поле было не что иное, как электрическое поле ».

А. Эйнштейн (1952), из письма к мемориалу Михельсона. Заседание Кливлендского физического общества, цитируемое Р.С. Shankland, Am. J. Phys., 32, 16 (1964), стр. 35.

Что Эйнштейн имел в виду под этим? Давайте внимательно рассмотрим пример.

Давайте внимательно посмотрим, что происходит, когда движущаяся заряженная частица отклоняется магнитной силой, создаваемой током в куске металлический провод. Во-первых, кусок провода сам по себе электрически нейтрален. Самый внешний электрон в металле может свободно перемещаться, поэтому мы можем думать провода в виде фиксированного массива положительных ионов металла, окруженного морем свободных электронов.Для простоты будем считать, что каждый атом металла вносит в море только один свободный электрон. Соседняя заряженная частица, взят, например, чтобы нести положительный заряд, не ощущает никакой силы от нейтральный провод, так как сила притяжения электронов нейтрализует из-за силы отталкивания от ионов металла. Когда электрический ток включается, свободные электроны начинают течь по проводу.

Рисунок 6. Диаграмма Минковского для ионов металла и свободных электронов. в проводе без тока.

В качестве полезного метода визуализации происходящего позвольте мне представить понятие диаграммы Минковского. Эти диаграммы названы в честь Гурвица. Минковский (1864–1909), обучавший молодого Эйнштейна в Цюрихском политехническом институте. в 1896 году, но позже внесла значительный вклад в математику. теории относительности. Диаграмма Минковского похожа на карту, которая дает обзор того, как объекты движутся во времени и пространстве. Например, рисунок 6 представляет ситуацию для нашего провода без тока в нем.Оба ионы металлов и свободные электроны неподвижны, поэтому всегда остаются на та же координата х . (Я пренебрегаю тепловыми эффектами, которые только заставляют ионы и свободные электроны беспорядочно перемещаться вокруг некоторого среднего положения.) Поэтому вертикальные линии, представляющие их положение как функцию времени, могут представляют позиции. Их называют мировыми линиями. По прошествии времени координаты x не меняются.

Теперь рассмотрим эффект включения электрического тока.Электроны начинают двигаться по проволоке с равномерной скоростью, v . Конечно, в реальном проводе электроны постоянно ускоряются и рассеивание ионов металлов, но общий эффект для равномерного дрейфовать по проводу под действием приложенного напряжения, которое преодолевает сопротивление провода.

Мы можем написать:

v = i / (e A N e )

, где v — скорость дрейфа электронов, i — ток в проводе, e — заряд электрона, A — площадь поперечного сечения провода и N e это число электронов на единицу объема.Мы также можем ввести величину, которая будет пригодится позже, названная линейной плотностью заряда, l :

l- = eAN e

Линейная плотность заряда — это просто количество свободного заряда электрона. за метр по проводу. Это будет отрицательное число, поскольку электроны несут отрицательный заряд, однако, поскольку мы предположили, что каждый атом металла вносит один электрон, это будет уравновешено равным и противоположным Плотность положительного заряда от ионов металлов, л + :

l- = l +

Мы знаем по опыту, что токоведущий провод генерирует магнитное поле. поле, которое оказывает магнитное воздействие на движущиеся заряженные частицы, но не на неподвижных заряженных частицах.Попробуем разобраться в происхождении эту магнитную силу, глядя на ситуацию с точки зрения движущегося заряда. Чтобы все было красиво и просто, предположим движущийся заряд имеет скорость v в том же направлении, что и движущийся электроны в проводе. Это была бы ситуация, если бы движущийся заряд двигались током по второму проводу.

Построим диаграмму Минковского для токоведущего провода. Мировые линии ионов металлов остаются прежними, так как они не движутся, однако мировые линии электрона наклоняются к ось x, поскольку координаты x электронов увеличиваются со временем (см. рисунок 7).В момент времени t 1 после того, как ток был переключен на, координата x электрона увеличилась на величину vt 1 .

Рис. 7. Электроны начинают двигаться, но ионы металлов остаются. фиксированный.

Теперь мы можем отметить на этой диаграмме систему отсчета ближайшего стационарного заряженная частица. Референсная рамка — это просто устройство, которое мы носим с собой. чтобы помочь нам воспринимать внешний мир.Восприятие внешнего мира включает в себя измерение расстояний и времени событий, происходящих вокруг нас. Мы можно просто нарисовать в системе отсчета неподвижной частицы с его оси x и t параллельны осям x и t, см. рисунок 8. Обратите внимание, что в новой рамке разделение между положительными ионами металлов и движение отрицательных свободных электронов такое же, как и раньше. Это может быть можно понять при внимательном рассмотрении влияния на электроны ускорение, которое они испытывают при включении тока.Поскольку расстояние между электронами не изменилось, положительные и отрицательные линейные плотности заряда снова имеют ту же величину, поэтому привлекательные и отталкивающие силы нейтрализуют друг друга, и неподвижная заряженная частица ощущает отсутствие электростатической силы:

л l + = L- l +

Мы уже знаем, что, несмотря на то, что ток электронов производит магнитное поле, неподвижная заряженная частица не чувствует магнитного поля сила.Формула силы Лоренца говорит нам, что магнитная сила равна нулю. если скорость равна нулю.

Рисунок 8: Система отсчета ближайшего стационарного заряженного частица.

Рассмотрим ближайшую движущуюся заряженную частицу. В этом случае Лоренц формула силы говорит нам, что такая движущаяся частица будет чувствовать магнитное усилие равное q v ´ В . Давайте сейчас посмотрим на ситуацию с точки зрения движущейся заряженной частицы.В своей собственной системе отсчета он неподвижен, v = 0, поэтому он не может чувствовать никаких магнитных сил, зависящих только от скорости. Это возможно, он чувствует какую-то другую силу? Будет полезно в нашей проверке этого, если мы отметим на нашей диаграмме Минковского систему отсчета движущаяся частица. Как выглядит его опорная рамка? Ну точка стационарная относительно начала координат системы отсчета движущихся частиц может быть представлен линией, подобной линиям для электронов.Я предполагаю, что движущаяся частица находилась в начале исходного кадра, когда ток был включен. Мировая линия происхождения движущегося frame также представляет ось t подвижной системы отсчета по определению (см. рисунок 9).

Рисунок 9: Ось времени системы отсчета движущегося электроны.

Построить ось x немного сложнее. Помните, что ось абсцисс просто очерчивает цепочку одновременных событий, которые произошли в t = 0 .Если мы сможем найти два одновременных события в движущейся системе отсчета, мы можно найти ось абсцисс. Для этого предположим, что один из электронов каким-то образом испускает вспышку света при t = 0 , как показано на рисунке 10.

Рисунок 10: Электрон испускает вспышку света, которая излучает в обоих направлениях.

Мы можем определить два одновременных события в движущейся рамке как события представленный получением этой вспышки двумя равноудаленными соседними электроны.Они отмечены двумя крестиками на рисунке 11. Помните. что электроны не знают, что они движутся, они видят, что положительный ионы металлов движутся назад мимо них.

Рис. 11: Крестиками отмечены одновременные события в электроне. система отсчета.

Теперь нам нужен необыкновенный физический элемент, чтобы посмотреть, что произойдет. следующий. Одна из самых удивительных вещей в том, как устроена Вселенная, — это что скорость света одинакова для всех наблюдателей.Как бы быстро вы идете, скорость света всегда остается той же! Это было главным озарением Эйнштейна в 1905 году и является фундаментом всей оптики и электромагнетизма. отдыхает.

Скорость света одинакова во всех системах отсчета, независимо скорости источника. Теперь, поскольку электроны равноудалены расположенные вдоль оси x, получение световых вспышек представляет собой одновременное события в их движущейся системе отсчета. Следовательно, ось x — это просто линия через два креста, начиная с общего начала.Обратите внимание, что события отмеченные двумя крестиками, не совпадают в исходной ссылке Рамка.

Рисунок 12: Система отсчета движущегося заряда наложена на системе отсчета ионов металлов.

Обратите внимание на поразительную вещь: мировые линии электронов пересекают ось x. с более широким интервалом по сравнению с мировыми линиями ионов металлов! Эти точки отмечены зелеными и розовыми кружками на рисунке 13. В справочнике кадра движущегося заряда, плотность заряда электронов меньше чем у ионов металлов! Теперь у нас есть:

l- + Рисунок 13: Линейные плотности заряда электронов и ионы металлов различны в электронном каркасе.

Следовательно, силы притяжения и отталкивания больше не уравновешены, что приводит к чистой электростатической силе, действующей на заряженную частицу.

Если мы внимательно проведем алгебру, необходимую для преобразования этого электростатического силы обратно в исходную систему отсчета ионов металла, в которой соседняя заряженная частица движется, мы находим, что она равна магнитная сила, которую мы ожидаем найти! Другими словами, что движется заряженная частица действует как чисто электростатическая сила от несбалансированной линейные плотности заряда описываются в исходной системе отсчета как сила, зависящая от скорости, которую мы называем магнитной силой.

Неуравновешенность линейных плотностей заряда между положительными металлами. ионов и движущихся электронов, измеренные в системе отсчета движущегося заряда, является результатом лоренцевского сжатия из-за относительных движений соседней заряженной частицы, электроны, текущие в проводе и ионы металлов. Этот релятивистский эффект, пожалуй, наиболее известен нам, когда применяется к быстро движущимся объектам. Посмотрим, как быстро движутся электроны в типичном токоведущем проводе.В медной проволоке плотность меди атомов примерно 8,5´10 22 атомов на кубический сантиметр, и, следовательно, плотность свободных электронов составляет около тоже самое. В медном проводе сечением 1 квадратный миллиметр и проводя ток 10 ампер, формула для v, приведенная выше, показывает, что скорость электронов составляет всего 0,7 миллиметра в секунду. Это чрезвычайно малая скорость! Лоренцево сжатие для такой малой скорости отличается от 1 только на 3´10 -24 .Этот невообразимо небольшого сжатия, тем не менее, достаточно, чтобы вызвать легкое дисбаланс в положительной и отрицательной плотности заряда провода, который заставляет движущиеся заряженные частицы чувствовать магнитную силу.

Имейте в виду, что эта магнитная сила намного слабее, чем любая из двух, почти уравновешенных, электростатических сил со стороны электронов или ионы металлов. Если бы свободные электроны с 1 метра провода могли быть полностью отделены от положительных ионов металла на 10 сантиметров, тогда притягивающие электростатические силы между этими двумя глыбами отрицательного и положительные заряды были бы примерно равны гравитационной силе между Земля и Луна! Это огромная сила электростатического силы, поэтому мы не часто используем их непосредственно в наших технологических Приложения.Разделить положительные и отрицательные заряды просто слишком сложно. Намного легче использовать невероятно небольшой дисбаланс, вызванный релятивистским лоренцевым сжатием, заметным как магнетизм.

Подумайте об этом в следующий раз, когда почувствуете таинственное притяжение магнита.

9.1: Введение в магнитный потенциал

  1. Последнее обновление
  2. Сохранить как PDF
  1. Соавторы

Нам знакома идея, что электрическое поле \ (\ textbf {E} \) может быть выражено как минус градиент потенциальной функции \ (V \).То есть

\ [\ textbf {E} = — \ textbf {grad} V = — \ nabla V. \ label {9.1.1} \]

Обратите внимание, что \ (V \) не уникален, потому что к нему можно добавить произвольную константу. Мы можем определить уникальный \ (V \), присвоив определенное значение \ (V \) некоторой точке (например, нулю на бесконечности).

Можно ли выразить магнитное поле \ (\ textbf {B} \) аналогично градиенту некоторой потенциальной функции \ (ψ \), например, \ (\ textbf {B} = — \ textbf {grad} \, ψ = — \ nabla ψ \)? Прежде чем ответить на этот вопрос, отметим, что есть некоторые различия между \ (\ textbf {E} \) и \ (\ textbf {B} \).В отличие от \ (\ textbf {E} \), магнитное поле \ (\ textbf {B} \) составляет без источника ; нет источников или стоков; силовые линии магнитного поля представляют собой замкнутые контуры. Сила, действующая на заряд \ (q \) в электрическом поле, равна \ (q \ textbf {E} \), и она зависит только от того, где находится заряд в электрическом поле, то есть от его положения. Таким образом, сила консервативная , и мы понимаем из любого исследования классической механики, что только консервативные силы могут быть выражены как производная от потенциальной функции.Сила, действующая на заряд \ (q \) в магнитном поле , равна \ (q \ textbf {v} \ times \ textbf {B} \). Эта сила (сила Лоренца) зависит не только от положения частицы, но и от ее скорости (скорости и направления). Таким образом, сила не консервативна. Это говорит о том, что, возможно, мы не можем выразить магнитное поле просто как градиент скалярной потенциальной функции — и это правильно; мы не можем.

Закон о силе Лоренца — Дополнительная документация SIMION 2020

Закон силы Лоренца описывает силу на точке зарядки q в электрическом ( E ) и / или магнитном ( B ) поле:

Траектория точечного заряда в электромагнитном поле можно определить с помощью этого уравнения вместе со вторым законом Ньютона ().

Обычно для расчета траектории частицы, такие как электроны и ионы, рассматриваются как точечные заряды, хотя в частицы практики могут иметь вращательную, колебательную или внутреннюю энергию, и диполи в частицах могут даже испытывать разные поля (например, как Википедия: Диэлектрофорез).

По умолчанию движение частиц в SIMION регулируется закон силы. Однако вы можете использовать язык сценариев SIMION для изменения или замените это уравнение на все, что захотите.Это включает добавление сила тяжести, применяющая силу вязкого сопротивления в соответствии с Уравнение мобильности, добавление диффузии или эффекты модели столкновения ионов с газом и т. д.

Гравитация

Вот наглядный пример пользовательской программы рабочего места, которая добавляет к частице силу (фактически ускорение) гравитации. Это ускорение добавляется к ускорению. электрического и / или магнитного поля, уже рассчитанного SIMION (из закона силы Лоренца):

 simion.workbench_program ()

- Добавьте ускорение свободного падения к вектору ускорения.2.
  ion_ay_mm = ion_ay_mm + g
конец
 

В большинстве случаев гравитация незначительна, если только частицы не имеют очень малую скорость, что может произойти с очень массивными частицами. Протон, ускоренный на 100 В, будет иметь скорость около 138 мм / мксек, что будет мало затронутым значением g на расстоянии 138 мм или около того. Такие массы, напряжения и расстояния типичны для многих настольных размеров. системы оптики частиц.

Разгон

Ускорение, наблюдаемое частицей в соответствии с законом силы Лоренца. зависит от отношения заряд / масса (q / m) частиц, а не от индивидуальные значения массы и заряда.Это видно, если сравнить закон силы Лоренца с чтобы получить .

Фактически, вы даже можете ввести произвольные значения массы и заряда в SIMION, если соотношение заряда / массы соответствует вашим реальным частицам и вы определяете / интерпретируете KE частицы с точки зрения введенной массы поскольку (или просто используйте скорость, а не KE). До SIMION 8.1.1.31 (в котором были увеличены пределы массы, заряда и KE), частица с массой более 1E + 9 u не была легко введена, но вышеприведенный трюк позволил его преодолеть.Другой вариант заключался в том, чтобы определить сегмент Accel_adjust в пользовательской программе. который при необходимости изменяет масштаб вектора ускорения, вычисленного SIMION. Просто для иллюстрации, чтобы изменить масштаб вычисленного ускорения SIMION в 10 раз:

.
 simion.workbenc_program ()
функция segment.accel_adjust ()
  местный масштаб = 10
  ion_ax_mm = ion_ax_mm * масштаб
  ion_ay_mm = ion_ay_mm * масштаб
  ion_az_mm = ion_az_mm * масштаб
конец
 

Калькулятор силы Лоренца

Вы можете использовать этот калькулятор силы Лоренца, чтобы вычислить влияние магнитного поля на заряженные частицы.Прочитав текст, вы узнаете о законе силы Лоренца, соответствующем уравнении силы Лоренца, а также о применении силы Лоренца в повседневной жизни.

Закон о силе Лоренца

Сила Лоренца возникает как общий эффект электрического и магнитного полей, действующих на заряженную частицу. Мы сосредоточимся здесь только на магнитной части силы. Для электрической части вы можете проверить Калькулятор закона Кулона.

Закон силы Лоренца гласит, что магнитное поле действует на движущуюся заряженную частицу.Величина силы зависит от заряда, скорости и силы магнитного поля. Что немного особенным, так это то, что направление силы не совпадает ни с траекторией движения частицы, ни с магнитным полем. Направление силы Лоренца перпендикулярно как направлению частицы, так и магнитному полю. В результате траектория частицы искривляется в магнитном поле. Кроме того, сила Лоренца равна нулю, если частица движется точно по линиям магнитного поля.Как мы можем записать эти наблюдения в математической форме? Это уравнение силы Лоренца.

Уравнение силы Лоренца

F = q v B sin (α)

где

  • q — заряд частицы,
  • v — скорость частицы,
  • B — напряженность магнитного поля,
  • α — угол между направлением траектории частицы и направлением магнитного поля,
  • F — результирующая сила.

В нашем калькуляторе силы Лоренца для простоты мы установили угол α = 90 ° . Если вы хотите его изменить, перейдите в расширенный режим. Вы можете видеть, что сила максимальна для этого угла, и если мы установим α = 0 ° , она также будет равна 0.

Lorentz Force Applications

Lorentz Force находит применение во многих областях. В науке он используется для ускорения частиц в циклотронах в поисках фундаментальных законов физики элементарных частиц. Он также используется в масс-спектрометрах, которые позволяют идентифицировать атомы и молекулы.Практические повседневные применения включают электродвигатели, громкоговорители и, что, вероятно, менее распространенные среди обычных домашних хозяйств, рельсотроны.

Что такое сила Лоренца? — Новости о хранении энергии, батареях, изменении климата и окружающей среде

Сила Лоренца — это закон физики, в частности, электромагнетизма, который описывает силовое взаимодействие между магнитными полями двух заряженных частиц. Он был назван в честь Хендрика Лоренца, голландского физика 1800-х годов, который проявлял большой интерес к науке об электромагнетизме.

Сила Лоренца лучше всего описывается уравнением

F = q (E + v x B)

, где B — напряженность магнитного поля, E — электрическое поле, q — заряд частицы и v — скорость частицы. В качестве единицы измерения он выражается в «Ньютонах».

Согласно этому уравнению, если частица с зарядом q движется (через другое магнитное поле) со скоростью v в присутствии электрического поля E и магнитного поля B , то она будет испытывать реактивную реакцию. сила.Эта реактивная сила известна как сила Лоренца .

Уравнение силы Лоренца фактически было получено из знаменитого уравнения Максвелла. Есть варианты этого основного уравнения силы Лоренца. Другие уравнения, которые можно найти в других учебниках, описывают магнитную силу на проводе с током, электродвижущую силу в проволочной петле, движущейся через магнитное поле, и силу, действующую на заряженную частицу, которая может двигаться со скоростью, близкой к скорости света. .

Другой интересный факт, обнаруженный уравнением силы Лоренца, — это направление магнитного поля, движения электронов и самой силы Лоренца. Это широко известно как «Правила рук», которые легко понять на этих фотографиях.

Когда заряд движущейся частицы отрицательный, используется правило левой руки .

Правило левой руки

Когда заряд движущейся частицы положительный, используется правило правой руки .

Правило правой руки

Изображения Источник: CK-12 Website

В наших предыдущих двух экспериментах на тему «Как сделать электромагнит?» и «Как магнитное всасывание работает на дверных звонках», мы поняли, как возникает электромагнетизм. Когда электрический ток наводится на провод, атомы начинают становиться движущимися зарядами (положительными или отрицательными в зависимости от протекания тока), которые действуют как маленькие магниты, указывающие в разных направлениях.

alexxlab

Добавить комментарий

Ваш адрес email не будет опубликован. Обязательные поля помечены *